Sei sulla pagina 1di 61

[Type text]

Electrochemical Cell
An electrochemical cell is a system consisting of electrodes that dip into an electrolyte and in
which a chemical reaction either uses or generates an electric current.

A voltaic or galvanic cell is an electrochemical cell in which a spontaneous reaction generates an


electric current.

A voltaic cell consists of two half-cells that are electrically connected. Each half cell is the
portion of an electrochemical cell in which a half-reaction take place.

A simple half-cell can be made from a metal strip that dips into a solution of its metal ion. An
example is the zinc-zinc ion half-cell (often called simply a zinc electrode), which consists of a
zinc metal strip dipping into a solution of a zinc salt. Another simple half-cell consists of a
copper metal strip dipping into a solution of a copper salt (copper electrode).

In a voltaic cell, two half-cells are connected in such a way that electrons flow from one metal
electrode to another through an external circuit, and ions flow from one half-cell to another
through an internal cell connection. Figure given below illustrates an atomic view of a voltaic
cell consisting of a zinc electrode and a copper electrode. As long as there is an external circuit,
electrons can flow through it from one electrode to another. Because zinc tends to lose electrons
more readily than copper, zinc atoms in the zinc electrode lose electrons to produce zinc ions.
These electrons flow through the external circuit to the copper electrode, where they react with
the copper ions to produce zinc ions and copper metal, and an electric current flows through the
external circuit.

The two half-cells must be connected internally to allow ions to flow between them. As zinc ions
continue to be produced, the zinc ion solution begins to build up a positive charge. Similarly, as
copper ions plate out as copper, the solution builds up a negative charge. The half cell reactions
will stop unless positive ions can move from the zinc half-cell to the copper half cell, and
negative ions from the copper half-cell can move to the zinc half-cell. It is necessary that these
ion flow occur without mixing of the zinc ion and copper ion solutions. If copper ion comes in
contact with the zinc metal, for example, direct reaction would occur without an electric current
being generated. The voltage would drop, and the battery would run down quickly.
A

Cl–, K+,
K+, Cl–,

LHE RHE
(Anode) (Cathode)

Zn++ Cl– K+, Cu++

[Type text]
[Type text]

The two half-cells of a voltaic cell are connected by a salt bridge. A salt bridge is a tube of an
electrolyte in a gel that is connected to the two half-cells of a voltaic cell; the salt bridge allows
the flow of ions but prevents the mixing of the different solutions that would allow direct reaction
of the cell reactants. The half-cells are connected externally so that an electric current flows.

The two half-cell reactions, as noted earlier, are


Zn(s) → Zn2+(aq) + 2e– (oxidation half-reaction)
Cu2+(aq) + 2e– → Cu(s) (reduction half-reaction)

The first half-reaction, in which a species loses electrons, is the oxidation half-reaction. The
electrode at which oxidation occurs is called the anode. The second half-reaction, which a species
gains electrons, is the reduction half-reaction. The electrode at which reduction occurs is called
the cathode. These definitions of anode and cathode hold for all electrochemical cells, including
electrolytic cells.

Note that the sum of the two half-reaction

Zn(s) + Cu2+(aq) → Zn2+(aq) + Cu(s)

It is the net reaction that occurs in the voltaic cell, it is called cell reaction.

Once you know which electrode is the anode and which is the cathode, you can determine the
direction of electron flow in the external portion of the circuit. Electrons are given up by the
anode (from the oxidation half-reaction) and thus flow from it, whereas electrons are used up by
the cathode (by the reduction half-reaction) and so flow into this electrode. The anode in a voltaic
cell has a negative sign, because electrons flow from it. The cathode in a voltaic cell has a
positive sign.

Note: 1. The salt-bridge contains solution of strong ionic salts like NaCl, NaNO3, KNO3, KCl etc., which is
soaked in a colloidal solution of agar-agar gel which permits the movement of ions of salts only.
2. It maintains the electrical neutrality of the solutions in the two half-cells. In the absence of salt bridge, a
reverse potential difference is set up in the two half-cells which results in breaking the continuous
supply of voltage.

Cell Notation: The efficiency of a cell depends on the tendency of LHE to lose electrons to
RHE and the ability of RHE to receive electrons. Cell efficiency is measured in terms of emf
(electromotive force), which is the difference in potentials of two electrodes due to which
electrons flow from anode to cathode. EMF value of a cell is constant for given concentration of
electrolyte, temperature and the pressure (in case of gases). Therefore, it depends on
concentration, temperature and pressure.

EMF of a cell is measured in terms of the difference in electric potential (electrical pressure)
between anode and cathode. Its unit is V(volt). The potential of half-cell is known as electrode
potential, which is the tendency of an electrode to get oxidized or reduced. Thus, tendency to get
oxidized is called oxidation potential and the tendency to get reduced is called reduction
potential.

[Type text]
[Type text]

Cell Notation: A cell can be represented as

LHE (Anode) || RHE (Cathode)

The anode or oxidation half-cell is always written on the left and the cathode or reduction half-
cell is written on the right. The two electrodes are electrically connected by means of a salt
bridge shown by two vertical bars.
2+ 2+
Zn (s) | Zn (aq) || Cu (aq) | Cu (s)
anode salt bridge Cathode

The half-cell which has higher reduction potential, acts as cathode and the electrode with lower
reduction potential or higher oxidation potential acts as anode.

The cell terminals are at the extreme ends in this cell notation, and a single vertical bar indicates
a phase boundary – say between a solid terminal and the electrode solution. For the anode of the
same cell, you have
2+
Zn (s) | Zn (aq)
anode terminal phase boundary anode electrolyte

When the half-reaction involves a gas, an inert material such as platinum serves as a terminal and
as an electrode surface on which the half-reaction occurs. The platinum catalyzes the half-
reaction but otherwise is not involved in it. Hydrogen bubbles over a platinum plate that is
immersed in an acidic solution. The cathode half-reaction is
+
2H (aq ) H 2(g)

The notation for the hydrogen electrode, written as a cathode, is


+
H (aq) | H 2(g) | Pt

To write such an electrode as an anode, you simply reverse the notation:


+
Pt | H 2(g) | H (g)
Here are several additional examples of this notation for electrodes (written as cathodes). A
comma separates ions present in the same solution. We will write the cathode with oxidized
species before reduced species, in the same order as in the half-reaction.

Cathode Cathode Reaction


Cl2(g) | Cl(aq) | Pt Cl2(aq) + 2e − −
2Cl(aq)
+ +
Fe3(aq) 2+
, Fe(aq) | Pt Fe3(aq) + e− 2+
Fe(aq)
+ 2+
Cd (aq) | Cd (s) Cd (aq) + 2e− Cd (s)

[Type text]
[Type text]

You can write the overall cell reaction from the cell notation by first writing the appropriate half-
cell reactions, multiplying as necessary, and then summing so that the electrons cancel/

Illustration 1: Writing the cell Reaction from the Cell Notation


a) Write the cell reaction for the voltaic cell
Tl(s) | Tl+(aq) || Sn2(aq) | Sn(s)
b) Write the cell reaction for the voltaic cell
Zn(z) | Zn2+(aq) || Fe3+(aq), Fe2+(aq) | Pt
Problem strategy: The cell notation gives the species involved in each half-
reaction. Complete the balance each half-reaction, then multiply by factors so that
when you add the half-reactions, the electrons cancel. The result is the cell
reaction.

Solution: a) The half-cell reactions are


Tl(s) → Tl+(aq) + e–, Sn2+(aa) + 2e– → Sn(s)
Multiplying the anode reaction by 2 and then summing the half-cell reactions
gives
2Tl(s) + Sn2+(aq) → 2Tl+(aq) + Sn(s)
b) The half cell reactions are
Zn(s) → Zn2+(aq) + 2e–. Fe3+(aq) + e– → Fe2+(aq) and the cell reaction is
Zn(s) + 2Fe3+(aq) → Zn2+(aq) + 2Fe2+(aq)
To fully specify the voltaic cell, it is necessary to give the concentrations of
solutions or ions and the pressure of gases. In the cell notation, these are written
within parentheses for each species.
For example, Zn(s) | Zn2+ (1.0 M) || H+ (1.0 M) | H2 (1.0 atm) | Pt

Nature of Electrodes: The various types of electrodes used and the chemical reactions which
take place are:

Gas Electrodes: Here the inert electrode is present in contact with the gas at 1 atm pressure
(unless specified) and its ions in the solution e.g. Hydrogen gas electrode, etc. It is denoted as
1
H+ | H2 (1 atm) | Pt and the chemical reaction is H+ + e– → H2(g).
2

The nature of inert electrode should be such that, when the external voltage is changed even by
small amount from the equilibrium value, the reaction will proceed one way or the other. Most
suitable electrode is platinum with fine coat of platinum powder or platinum black.

Oxidation – Reduction Electrode: Here the inert electrode (generally platinum) is dipping
in the solution which contains ions of species having two oxidation states, e.g. platinum electrode
dipping in solution containing stannous and stannic. It is denoted as Sn4+, Sn2+ | Pt and chemical
reaction is Sn4+ + 2e– → Sn2+. The solution can be a neutral species present in two oxidation
stages e.g. H+, Q, QH2 | Pt.

[Type text]
[Type text]

Where QH2 is the hydroquinone an Q is quinine. It is commonly known as quinhydrone


electrode. The chemical reaction is
O OH

+2H + + 2e− 

O OH

Meta-Metal Ion Electrode: Here the metal electrode dips in a solution containing its ions and
the metal electrode takes part in chemical reaction. Reactivity of the metal should be intermediate
otherwise the reactive metals will react with water and cannot act as electrodes. The example is
the copper electrode dipping in copper sulphate solution. It is denoted as Mn+ | M in general and
in particular Cu2+ (c) | Cu and c is the concentration. The chemical reaction is
Cu2+ + 2e– → Cu(s).

Metal-Insoluble Salt Anion Electrode: Here the metal electrode is in contact with its ions
(generally the anion). The most common is the calomel electrode. This consists of mercury,
mercurous chloride (Hg2Cl2) and saturated solution of KCl. But the concentration of Cl– ions can
be different. The chemical reaction taking place is

1
Hg 2Cl 2(s) + e − 
→ Hg (s) + Cl− and Cl− | HgCl 2 | Hg | Pt
2
Similarly lead amalgam lead sulphate electrode is depicted as SO 24− | PbSO 4 | Pb(Hg) and the
chemical reaction is PbSO4(s) + 2e− 
→ Pb(s) + SO 24−

Illustration 2: Write cell electrode for the following reactions


a) 2MnO4– + 6H+ + 5H2C2O4 2Mn2+ + 8H2O + 10CO2(g)
b) Cu(OH)2(s) → Cu2+ + 2OH–

Solution: a) Half-cell reactions are


MnO −4 + 8H + + 5e−  → Mn 2+ + 4H 2O
2CO 2 + 2H + + 2e − 
→ H 2CO 4
MnO 4− , H + , Mn 2+ | Pt and Pt|CO2 | H +

b) Half-cell reactions are


Cu(OH)2(s) + 2e−  → Cu + 2OH −
Cu 2+ + 2e− 
→ Cu (s) and electrodes are
Cu 2+ | Cu and Pt | Cu(OH)2 | OH −

Standard Cell EMF’s and Standard Electrode Potential

[Type text]
[Type text]

A cell emf is a measure of the driving force of the cell reaction. This reaction occurs in the cell as
separate half-reactions: an oxidation half-reaction and a reduction half-reaction. The general
forms of these half-reactions are

Reduced species → oxidized species + ne– (oxidation/anode)


Oxidized species + ne– → reduced species (reduction/cathode)
Ecell = oxidation potential + reduction potential

If you can construct a table of reduction potentials, you will have a list of strengths of oxidizing
agents, in addition to having a way of calculating cell emf’s.

You can look at an oxidation half-reaction as the reverse of a corresponding reduction half-
reaction. The oxidation potential for an oxidation half-reaction equals to negative of the reduction
potential for the reverse half-reaction (which is a reduction).
Oxidation potential for a half-reaction = reduction potential for the reverse half-reaction.
This means that in practice you need to tabulate only oxidation potential or reduction potentials.
The choice, by convention, is to tabulate reduction potentials. We call these electrode potentials,
and we denote them by the symbol E (without the subscript cell, as in Ecell).
2+ 2+
Zn (s) | Zn (aq ) || Cu (aq) | Cu (s)

The half-reaction are Zn(s) → Zn2+(aq) + 2e–; Cu2+(aq) + 2e– → Cu(s)


The first half-reaction is an oxidation. If you write EZn for the electrode potential corresponding
to the reduction half-reaction Zn2+(aq) → Zn(s), then –EZn is the potential for the oxidation half-
reaction Zn(s) → Zn2+(aq) + 2e–. The copper half-reaction is a reduction. Write ECu for the
electrode potential.
The cell emf is the sum of the potentials for the reduction and oxidation half-reactions. For the
cell we have been describing, the emf is the sum of the reduction potential (electrode potential)
for the copper half-cell and the oxidation potential (negative of the electrode potential) for the
zinc half-cell.

E cell = E Cu + (−E Zn ) = E Cu − E Zn

Note that the cell emf equals the difference between the two electrode potentials. You can think
of the electrode potential as the electric potential on the electrode, and you obtain the cell emf as
a potential difference in which you subtract the anode potential from the cathode potential.
Ecell = Ecathode – Eanode
The electrode potential is an intensive property. This means that its value is independent of the
amount of species in the reaction. Thus, the electrode potential for the half-reaction.

2Cu2+(aq) + 4e– → 2Cu(s) is the same as for Cu2+(aq) → Cu(s)

[Type text]
[Type text]

Note: 1. If Ecell is positive, then the cell reaction is spontaneous


2. If in a cell reaction, more than one electron is involved, the standard electrode potential does not
0
change but the standard free energy of the chemical reaction will be multiple of FE , i.e. then actual
value of n should be used.

Tabulating Standard Electrode Potential: The emf of a voltaic cell depends on the
concentrations of substances and the temperature of the cell. For purposes of tabulating
electrochemical data, it is usual to choose thermodynamic standard-state conditions for voltaic
cells. The standard emf, E0cell, is the emf of a voltaic cell operating under standard-state
conditions (solute concentrations are each 1 M, gas pressures are each 1 atm, and the temperature
has a specified value – usually 25°C). Note the superscript degree sign (0), which signifies
standard-state conditions.

If you can derive a table of electrode potentials, you can calculate cell emf’s from them. This
provides a great advantage over tabulating cell emf’s. From a small table of electrode potentials,
you can obtain the emf’s of all the cells that you could construct from pairs of electrodes. For
instance, a table of 40 electrode potentials would give the emf’s of nearly 800 voltaic cells.

However, it is not possible to measure the potential of a single electrode; only emf’s of cells can
be measured. What you can do is measure the emf’s of cells constructed from various electrodes
connected in turn to one particular electrode, which you choose as a reference. You arbitrarily
assign this reference electrode a potential equal to zero and obtain the potentials for the other
electrodes by measuring the emf’s. By convention, the reference chosen for comparing electrode
potentials is the standard hydrogen electrode.

The standard electrode potential, E0, is the electrode potential when the concentrations of solutes
are 1M, the gas pressures are 1 atm, and the temperature has a specified value (usually 25°C).
The superscript degree sign (0) signifies standard state conditions.

To understand how standard electrode potentials are obtained, look at how you would find the
standard electrode potential. E0, for the zinc electrode. You connect a standard zinc electrode to a
standard hydrogen electrode. When you measure the emf of the cell with a voltmeter, you obtain
0.76V, with the zinc electrode acting as the anode.

Now write the cell emf in terms of the electrode potentials.

The cell is Zn(s) | Zn2+(aq) || H+(aq) | H2(g) | Pt and the half-reactions with corresponding half-cell
potentials (oxidation or reduction potentials ) are Zn(s) → Zn2+(aq) + 2e– ; – E 0Zn

The cell emf is the sum of the half-cell potentials. E cell = E 0H2 + (− E 0Zn )

Substitute 0.76V for the cell emf and 0.00V for the standard hydrogen electrode potential. This
gives E 0Zn = −0.76V.

Proceeding in this way, you can obtain the electrode potential for a series of half-cell reactions.
Table given below lists standard electrode potentials for selected half-cells at 25°C.

[Type text]
[Type text]

Cathode (Reduction) Half Reaction Standard Potential E(V)


+ − – 3.04
Li (aq) +e Li (s)
+
Na (aq) + e− Na (s) – 2.71
2+
Mg (aq) + 2e − Mg (s) – 2.38

Al3+ (aq) + 3e− Al(s) – 1.66

2H 2O(l) + 2e − −
H 2(g) + 2OH (aq) – 0.83
2+
Zn (aq) + 2e− Zn (s) – 0.76
3+
Cr(aq) + 3e − Cr(s) – 0.74
2+
Fe(aq) + 2e − Fe(s) – 0.41
2+
Cd (aq) + 2e− Cd (s) – 0.40
2+
Ni (aq) + 2e− Ni (s) – 0.23
2+
Sn (aq) + 2e − Sn (s) – 0.14
2+
Pb (aq) + 2e− Pb (s) – 0.13
2+
Fe(aq) + 2e − Fe(s) – 0.04
+
2H (aq) + 2e− H 2(g) 0.00
4+ − 2+ 0.15
Sn (aq ) + 2e Sn (aq)
2+ − + 0.16
Cu (aq ) +e Cu (aq)
2+ − 0.34
Cu (aq ) + 2e Cu (s)

IO(aq) + H 2O(l) + 2e − −
I(aq) −
+ 2OH (aq) 0.49
+ − 0.52
Cu (aq ) +e Cu (s)
I 2(s) + 2e − −
2I(aq )
0.54
+
Fe3(aq) + e− 2+
Fe(aq) 0.77
+ − 0.80
Hg 22(aq ) + 2e 2Hg (aq )
2+
Ag (aq) + 2e − 2Ag (l) 0.80
+
Ag (aq) + e− Ag (l) 0.85

ClO(aq) + H 2O(l) + 2e − −
Cl(aq) −
+ 2OH (aq) 0.90
2+
Hg (aq) + 2e − Hg (aq) 0.90

[Type text]
[Type text]

− +
NO3(aq) + 4H (aq) + 3e− NO(g) + 2H 2O(l) 0.96

Br2(l) + 2e− −
2Br(aq) 1.07
+
O2(g) + 4H (aq) + 4e − 2H 2O(aq) O 1.23
−2 +
Cr2 O7(aq) + 8H (aq) + 6e − 3+
2Cr(aq) + 7H 2 O(l) 1.33

Cl2(g) + 2e− −
2Cl(aq) 1.36

MnO −4(aq) + 8H (aq)


+
+ 6e− 2+
Mn (aq) + 4H 2O(l) 1.49
+
H 2O 2(aq) + 2H (aq) + 2e − 2H 2O(l) 1.78
2−
S2O8(aq) + 2e − 2−
2SO 4(aq) 2.01

F2(g) + 2e − −
2F(aq) 2.87

Strength of Oxidising and Reducing Agents: Standard electrode potentials are useful
in determining the strengths of oxidizing and reducing agents under standard-state conditions.
Because electrode potentials are reduction potentials those reduction half-reactions in the table
with the larger (that is more positive) electrode potentials have the greater tendency to go left to
right as written. A reduction half-reaction has the general form.

Oxidised species + ne– → reduced species.

The oxidized species acts as an oxidizing agent. Consequently, the strongest oxidizing agents in a
table of standard electrode potentials are the oxidized species corresponding to half-reactions
with the largest (most positive) E0 values.

Those reduction half-reactions with lower (that is, more negative) electrode potentials have a
greater tendency to go right to left. That is,

Reduced species → oxidized species + ne–

The reduced species acts as a reducing agent. Consequently, the strongest reducing agents in a
table of standard electrode potentials are the reduced species corresponding to half-reactions with
the smallest (most negative) E0 values.

The first two and last two entries in the table are as follows:

+
Li(aq) + e− Li (s)
+
Na (aq) + e− Na (s)


2−
S2O8(aq) + 2e − 2−
2SO 4(aq)

[Type text]
[Type text]

F2(g) + 2e − −
2F(aq)

The strongest oxidizing agents are the species on the left side in the last two entries in the table.
The strongest reducing agents are the species on the right side in the first two entries in the table.

Standard EMF of a cell (E0)

E0 = difference in potentials of two half-cells


E0 = {standard oxidation potential of anode – standard oxidation potential of cathode}
E 0cell = E 0A − E 0C A: anode C: cathode ox: oxidation
ox ox

or
E0 = {standard reduction potential – standard reduction potential of anode}
E 0cell = E 0A − E C0 A: anode C cathode ox: oxidation

Illustration 3: Electrode potential of the metals in their respective solution are provided.
Arrange the metals in their increasing order of reducing power.
K+/K = – 2.93V, Ag+/Ag = + 0.80V
Hg+/Hg = +0.79V, Mg2+/Mg = – 2.37V
Cr3+/Cr = –0.74V

Solution: We know that the reducing power of a metal depends upon its tendency to lose
electrons. Thus lower the reduction potential, more the tendency to get oxidized
and thus more will be the reducing power. Hence increasing order of reducing
power is:
Ag < Hg < Cr < Mg < K

Illustration 4: Using the standard electrode potentials predict the reaction, if any, that occurs
between the following:
a) Fe3+(aq) and I–(aq) b) Ag+(aq) and Cu(s)
c) Fe3+(aq) and Br–(aq) d) Ag(s) and Fe3+(aq)
2+
e) Br2(aq) and Fe (aq)
Given: E 0Fe3+ / Fe2+ = +0.77V, E I / I− = +0.54V
2

E 0Ag+ / Ag = +0.80V, E Cu
0
2+
/ Cu
= +0.34V

E 0B − = +1.08V
2 / Br

Solution: a) Here I–(aq) loses electrons and Fe3+(aq) gains electrons. Thus
Oxidation half cell reaction 2I– → I2 + 2e–, E0 = – 0.54V
Reduction half cell reaction [Fe3+ + e– → Fe2+] × E0 = +0.77V
––––––––––––––––––––––––––––––––––––
Overall reaction 2I– + 2Fe3+ → I2 + 2Fe2+, E0cell = 0.23V
Since E 0cell is +ve, the reaction is spontaneous i.e., the reaction does take place.
b) Here Cu(s) loses electrons and Ag+(aq) gains electrons. Thus

[Type text]
[Type text]

Oxidation half cell reaction: Cu → Cu2+ + 2e–, E0 = – 0.34V


Reduction half cell reaction [Ag+ + e– → Ag] × 2, E0 = +0.80V
––––––––––––––––––––––––––––––––––––
Overall reaction Cu + 2Ag+ → Cu2+ + 2Ag, E0cell= 0.46V
Since E0cell is +ve, the reaction is spontaneous
c) Here Br–(aq) loses electrons and Fe3+(aq) gains electrons. Thus
Oxidation half cell reaction 2Br– → Br2 + 2e–, E0 = – 1.08V
Reduction half cell reaction 2Fe3+ + 2e– → 2Fe2+, E0 = +0.77V
––––––––––––––––––––––––––––––––––––
Overall reaction 2Br– + 2Fe3+ → Br2 + 2Fe2+, E0cell = 0.31V
0
Since E cell is –ve, the reaction is non-spontaneous, i.e., the reaction does not
take place.
d) Here Ag loses electrons and Fe3+(aq) gains electrons. Thus
Oxidation half cell reaction Ag → Ag+ + e–, E0 = – 0.80V
Reduction half cell reaction Fe3+ +e– → Fe2+, E0 = +0.77V
––––––––––––––––––––––––––––––––––––
Overall reaction Ag + Fe3+ → Ag+ + Fe2+, E0cell = – 0.03V
Since E0cell is –ve, the reaction is non-spontaneous, i.e., the reaction does not
take place.
e) Here Fe2+(aq) loses electrons and Br2(aq) gains electrons. Thus
Oxidation half cell reaction 2Fe2+ → 2Fe3+ + 2e–, E0 = – 0.77
Reduction half cell reaction Br2 + 2e– → 2Br–, E0 = +1.08V
––––––––––––––––––––––––––––––––––––
Overall reaction 2Fe2+ + Br2 → 2Fe3+ + 2Br–, E0cell = +0.31V
Since E0cell is +ve, the reaction is spontaneous, i.e., the reaction does not take
place.

Illustration 5: Calculate the values for cell


2+
Zn aq Cu 2aq+
Zn Cu
1M 1M
(i) cell reaction and (ii) e.m.f. of cell
Given: E 0Cu 2+ / Cu = +0.35V; E 0Zn 2+ / Zn = −0.76V

Solution: i) for Cu/Cu2+ = – 0.35V


E 0OP
2+
E 0OP for Zn/Zn = +0.76V
More is E 0OP , more is tendency to show oxidation and thus Zn will oxidize and
Cu2+ will reduce.
Anode Zn → Zn2+ + 2e–
Cathode Cu2+ + 2e– → Cu
––––––––––––––––––––––
Cell reaction Zn + Cu2+ → Zn2+ + Cu
ii) Also E 0cell = E 0OP 2+ + E 0RP 2+ = 0.76 + 0.35 = 1.11V
Zn / Zn Cu / Cu

[Type text]
[Type text]

Dependence of emf of a Cell on Concentration: The emf of a cell depends on the


concentration of ions and on gas pressures. For that reason, cell emf’s provide a way to measure
ion concentration. The pH meter, for example, depends on the variation of cell emf with
hydrogen ion concentration. You can relate cell emf’s for various concentrations of ions and
various gas pressures to standard electrode potentials by means of an equation first derived by the
German Walther Nernst (1864 – 1941).

Nernst Equation
Recall that the free-energy change,
∆G = ∆G ° + RT lnQ
Here Q is the thermodynamic reaction quotient. The reaction quotient has the form of the
equilibrium constant, except that the concentrations and gas pressures are those that exist in a
reaction mixture at a given instant. You can apply this equal to a voltaic cell. In that case, the
concentrations and gas pressures are those that exist in the cell at a particular instant. If you
substitute ∆G = – nFEcell and ∆G0 = – nFE0cell into this equation, you obtain.
− nFE cell = −nFE 0cell + RTlnQ

The result rearranges to give the Nernst equation, an equation relating the cell emf to its standard
emf and the reaction quotient.
RT 2.303RT
E cell = E 0cell − ln Q or E cell = E 0cell − log Q
nF nF

If you substitute 298K (25°C) for the temperature in the Nernst equation and put in values for R
and F, you get (using common logarithms).
0.0591
E cell = E cell
0
− log Q (values in volts at 25°C)
n

You can show from the Nernst equation that the cell emf, Ecell, decreases as the cell reaction
proceeds. As the reaction occurs in the voltaic cell, the concentrations of products increase and
the concentrations of reactants decrease. Therefore, Q and log Q increase. The second term in the
Nernst equation, (0.0592/n) log Q, increases, so that the difference E0cell – (0.0592/n) log Q
decreases. Thus, the cell emf, Ecell, becomes smaller. Eventually the cell emf goes to zero, and the
cell reaction comes to equilibrium.
For Galvanic Cell Zn(s) | Zn2+ (x1) || Cu2+ (x2) | Cu(s)
Let us assume Zn to be anode and Cu to be cathode, though this can be correct only when the
emf of the cell is positive.
Thus, Ecell = EC – EA

For cathode Cu2+ + 2e– → Cu(s)


For anode Zn2+ + 2e– → Zn(s)

[Type text]
[Type text]

0.059 1
Therefore E Cu 2+ / Cu = E 0Cu 2+ / Cu = log
2 [Cu 2+ ]
0.059 1
E Zn 2+ / Zn = E 0Zn 2+ / Zn − log
2 [Zn 2+ ]

Putting these values in(1), we get


0.059 [Zn 2+ ]
E cell = E cell
0
− log
2 [Cu 2+ ]

Electromotive Force
Potential difference is the difference in electric potential (electrical pressure) between two points.
You measure this quantity in volts. The volt, V, is the SI unit of potential difference. The
electrical work expended in moving a charge though a conductor is
Electrical work = charge × potential difference
Corresponding SI units for the terms in this equation are
Joules = coulombs × volts
The Faraday constant, F is the magnitude of charge on one mole of electrons; it equals 9.65 × 104
C (96.500 coulombs). The faraday is a unit of charge equal to 9.65 × 104C. In moving this
quantity of charge (one faraday of charge) from one electrode to another, the numerical value of
the work done by a voltaic cell is the product of the faraday constant F times the potential
difference between the electrodes. The work w is the negative of this, because the voltaic cell
loses energy as it does work on the surroundings.
w = – F × potential difference.
In the normal operation of a voltaic cell, the potential difference (voltage) across the electrodes is
less than the maximum possible voltage of the cell. One reason for this is that it takes energy or
work to drive a current through the cell itself. The decrease in cell voltage as current is drawn
reflects this energy expenditure within the cell; and the greater the current, the lower the voltage.
Thus, the cell voltage has its maximum value only when no current flows. The situation is
analogous to measuring the difference between the pressure of water in a faucet and that of the
outside atmosphere. The maximum potential difference between the electrodes of a voltaic cell is
referred to as the electromotive force (emf) of the cell, or Ecell.

We can now write an expression for the maximum work obtainable from a voltaic cell. Let n be
the number of electrons transferred in the overall cell equation. The maximum electrical work of
a voltaic cell for molar amounts of reactants (according to the cell equation as written) is
Wmax = nFEcell
Here Ecell is the cell emf, and F is the Faraday constant, 9.65 × 104C

Illustration 6: Calculate the standard e.m.f. of the reactionFe3+ + 3e– → Fe(s) E 03 = ?


Given Fe3+ + e– → Fe2+ E10 = 0.771V Fe2+ + 2e– → Fe(s) E 02 = – 0.44V

[Type text]
[Type text]

Solution: With the help of calculation of free energy


∆G10 = − nFE 0 = −(1)(F)(0.771V) = −0.771V
∆G 02 = −(2)(F)(0.44) = 0.88FV
∆G 30 = −(3)(F)(E 0 ) = −3Fe0 V
The free energy change for the unknown process can be obtained as
∆G 30 = ∆G10 + ∆G 02
−3FE 0 = −0.771F + 0.88F = 0.109F
0.109
or E 0 = − = −0.036V
3

Illustration 7: The standard electrode potentials of the electrode Cu2+|Cu and Ag+|Ag are 0.7991V.
What would be the concentration of Ag+ in a solution containing 0.06M of Cu2+ ion
such that both the metals can be deposited together. Assume that activity coefficients
are unity and both silver and copper do not dissolve among themselves.
Solution: The individual reactions are
Cu2+ + 2e– → Cu(s)
Ag+ + e– → Ag(s)
The electrode potentials given by Nernst equation
0.0591 1
E(Cu 2 + | Cu) = E 0 = log
2 [Cu 2 + ]
(0.0591) 1
= 0.037 – log = 0.037 – 0.036 = 0.301
2 0.06
0.0591 1
E(Ag + | Ag) = 0.7991 − log
1 [Ag + ]
Two metals will be deposited together when the electrode potentials are equal i.e.
1
0.301 = 0.7991 − 0.0591log
[Ag + ]
1 0.7991 − 0.301
i.e.log = = 8.428
[Ag + ] 0.0591
1
= 108.428
[Ag + ]
or [Ag+] = 10–8.428 = 0.37 × 10–8 mol dm–3

Thermodynamics of the Cells


The e.m.f. of the cell is related to free energy by equation(1) i.e.

∆G = –nFE …(1)
 ∂∆G 
Now  ∂T  = −∆S
 P
 ∂∆G   ∂E 
So  ∂T  = −nF  ∂T  = −∆S
 P  P

[Type text]
[Type text]

 ∂E 
or ∆S = nF   …(2)
 ∂T P
The enthalpy of the cell reaction will be
∂E 
∆H = ∆G + T∆S = − nFE + TnF   …(3)
 ∂T P

The thermodynamic quantities of the cell reaction can be calculated by equations (1), (2) and (3)
provided one knows the emf of cell and its dependence on temperature.

The heat effects in the system can be calculated as follows. If the process is irreversible (i.e. by
mixing the reactants together), heat flow to the system can be given by the reaction ∆H = QP. If
the process is reversible the heat flow to system is given by QP = T∆S.

Illustration 8: Consider the cell Cd | CdCl2⋅25H2O | AgCl(s) Hg


The emf of the cell at 15°C is 0.67531V and temperature coefficient of emf is 0.00065V
deg–1. Calculate the value of ∆H at 15°V and heat flow if the process is carried
reversibly.

Solution: The cell reaction is


At anode Cd(s) + 2Cl– → CdCl2(s) + 2e–
At cathode 2AgCl(s) + 2e– → 2Ag(s) + 2Cl–
–––––––––––––––––––––––––––
Cd(s) + 2AgCl(s) → CdCl2(s) + 2Ag(s)
The number of electrons involved are two
So, ∆G = – nFE
= – (2) (96500C) (0.67531V)
= – 130.335kJ
∆S = (2) (96500C) (– 0.0065V deg–1)
= – 125.45 J deg–1
∆H = ∆G + T∆S
= (–130.335 kJ) + (288 K) (– 125.45 JK–1) (10–3 kJ J–1)
= – 166.465kJ
QP = (288K) (–125.45 JK–1 mol–1) = 36.

Condition of Equilibrium (Ecell = 0.0V)

When Ecell = 0.0V, i.e., no potential difference is obtained between the terminals of cell (battery),
the cell reaction in such a state is said to be in equilibrium. So in such cases,

When Ecell = Q = Keq = equilibrium constant.


0.059
E cell = E 0cell − log Q
n
0.059
0.0 = E 0cell − log K eq
n

[Type text]
[Type text]

0.059
E 0cell = log K eq
n

RT a Ag + a Fe2+
i.e. E0 = ln ln K
nF a Fe3+

Since, ‘a’ for pure solid is unity.

0.0591
At 25°C the above equation becomes E0 = log K
n

(0.281)V(1)
i.e. log K = = −0.4751
(0.0591V)
K – 0.355

Illustration 9: Calculate the equilibrium constant for the reaction


Zn2+ + 4NH3 → [Zn(NH3)4]2+
E 0 (Zn2+/Zn) = – 0.763 and E 0 [Zn(NH3)4]2+/Zn + NH3) = 1.03V

Solution: The electrode reactions for the given electrodes, can be written as
Zn2+ + 2e– → Zn E10 = – 0.763V
[Zn(NH3)4]2+ + 2e– → Zn + 4NH3 E10 = – 1.03V
Reaction (1) – (2)
Zn2+ + 4NH3 → [Zn(NH3)4]2–
The standard emf of this reaction = E10 − E 02 = – 0.763V – (–1.03) = 0.267V

RT a[Zn ( NH3 )4 ]2+


According to Nernst equation E = E 0 − ln
2F a Zn 2+ ⋅ a

0.0591
If the process is equilibrium, E = 0 at 25°C log K = 0.267
2
(0.267)(2)
log K = = 9.036
(0.059)
K = 10−0.36109 = 1.09 × 109
The equilibrium constant is also called the “Stability Constant” of the complex. If
the reaction is written in a reverse manner, it will be “instability constant”.

Solubility Product
The solubility product of sparingly soluble salt MX can be discussed in terms of the equilibrium
of the kind

[Type text]
[Type text]

+ −
MX (s) M (aq) + X (aq)

Since the activity of the pure solid is always unity, the equilibrium constant of the solubility
product can be written.
K sp = a M + a x − …(1)

When the solubility of the salt is very small, the solution is ideal and thus a1 = c1 of ions. The
equation (1) can be written as
RT a M + a X −1
E = E0 − ln
nF a MX

Under equilibrium conditions the emf of the cell will be zero i.e., E = 0 and also the activity of
pure solid is unity. Further K sp = a M + ⋅ a X − , the above equation at 25°C can be written as

E0 ⋅ n
log K sp = …(2)
0.0591
The solubility product thus can be calculated from the standard emf of one cell, formed in such a
way that the final reaction is the type given above.

Illustration 10: Calculate the Ksp of AgI by forming proper cell.


Given E 0I− / Ag / Ag
= −0.151V and E 0Ag+ / Ag = 0.7991V
(s )

Solution: The cell can be written as Ag | Ag+ || I– | AgI | Ag


At left electrode Ag(s) → Ag+ + e– E0 = 0.7991V
At right electrode Ag(s) + e → Ag(s) + I
– –
E0 = – 0.151V
AgI(s) → Ag+ + I–
The standard emf of the cell is
E 0 = E 0R − E 0L = −0.151 − 0.7991 = – 0.9501V
Using relation (1) at 25°C
(0.9501)(1)
log K sp = − = −16.1
0.059
K sp = 10−16.1 = 7.94 × 10−17

Concentration Cells
In concentration cell, the two electrodes are of the same material and they are dipping in the
solutions of their respective ions at different concentrations. The two solutions are separated by
salt bridge. The electrode dipping in a solution of higher concentration of positive ions is positive
and the electrode dipping in lower concentration of positive ions is negative. The overall cell
reaction is nothing but a transference of material from higher activity to lower activity. The emf
of the cell is given by

[Type text]
[Type text]

RT a1
E=− ln
nF a 2

where a1 and a2 are the activities of the ion in two solutions. The term E0 cancels but because
both the electrodes are same and will have the same standard electrode potential at unit activity.
The E is positive if a2 > a1 and negative if a1 > a2.

Concentration Cell in which Electrodes is Reversible with Respect to Cation

a) Zn | Zn2+ (C1) || Zn2+ (C2) | Zn


At L.H.S. half cell Zn → Zn2+ (C1) + 2e– E0 = 0.76V
At R.H.S. half cell Zn2+ (C2) + 2e– → Zn E 0R = −0.76V
––––––––––––––––––––––––––––––––––––
Net Zn2+ (C2) → Zn2+ (C1) E 0cell = 0.00V
––––––––––––––––––––––––––––––––––––
0.0591 C 
E cell = E 0cell − log  1 
2  C2 
0.0591 C
E cell = log 2
2 C1
Cell reaction is spontaneous in forward direction if [Zn2+]RHS, (C2) > [Zn2+]LHS, (C1)
b) Pt (H2) | HCl (C1) || HCl (C2) | Pt (H2)
1 atm 1 atm
0.0591 C 
E cell = E 0cell − log  1 
2  C2 
C2
E cell = 0.0591log
C1

Concentration Cell in which Electrode is Reversible with Respect to Anion

Pt (Cl2) | Cl– (C1) || Cl– (C2) | Pt (Cl2)


1 atm 1 atm
0.0591 C 
E cell = E 0cell − log  1 
2  C2 
0.0591 C
E cell = log 1
1 C2
For such cases, cell reaction is spontaneous in forward reaction if C1 > C2.

Concentration Cell having Electrodes at Different Concentration Dipped into


Same Electrolyte

[Type text]
[Type text]

Pt(H 2 ) | HCl | Pt(H 2 )


at p1 1M at p 2

Two half cells are joined by two salt – bridges since they have common electrolyte
1
L.H.S. half cell H 2 (P1 )  → H + + e−
2
1
R.H.S. half cell H + + e −  → H 2 (P2 )
2
1M
––––––––––––––––––––
Net H2 (P1) → H2 (P2) E0cell = 0.00V
––––––––––––––––––––
0.0591  p 2 
Ecell = E 0cell − log  
2  p1 
p1
Ecell = 0.591 log
p2
Cell reaction is spontaneous in forward direction if p1 (L.H.S.) > p2 (R.H.S.)

Illustration 11: Calculate the emf of the cell Zn-Hg (C1)/Zn2+ (aq) | Zn – Hg (C2) at 25°C, if the
concentration of the zinc amalgam are C1 = 2 gm per 100 gm of Hg and C2 = 1 gm per
100 gm of Hg.
Solution: At cathode: Zn2+ + 2e– → Zn(C2)
At anode: Zn(C1) → Zn2+ + 2e–
––––––––––––––––––––––––––––––––
Net reaction Zn(C1) → Zn (C2)
0.059 C 1
∴ E= log 2 = −0.295log   = 8.8 × 10−3 V
2 C1 2
Electrolyte Concentration Cells: Two electrodes of the same metal are dipping in solutions
of metal ions of different concentrations.

e.g. Zn | Zn2+ (C1) || Zn2+ (C2) | Zn


A+ cathodes Zn2+ (C2) + 2e– → Zn(s)
A+ anode Zn(s) + 2e– → Zn2+ (C1) + 2e–
–––––––––––––––––––––––––––
Zn2+ (C2) → Zn2+ (C1)
RT C2
∴ E cell = ln
nF C1

For the process to be spontaneous, emf should be the. Hence, C2 > C1.

Electrolysis and Electrode Reactions: Electrolysis is a process in which chemical


reactions occur at the electrodes, dipping in the electrolytes, when voltage is applied across them.

[Type text]
[Type text]

The electrode that is charged positively is called anode and the electrode that is charged
negatively is called cathode. The electrodes, like platinum, which only transfer electrons to and
from the solutions are termed as “inert electrodes”. The “reactive electrodes” are those which
enter chemically into the electrode reactions. During electrolysis, the reduction takes place at the
cathode and oxidation takes place at the anode.

There can be many types of electrode reactions but general features are summarized below:

i) Current carrying ions are discharged at the electrodes.


ii) A negative ion which is difficult to get discharged at the anode, leads to the
decomposition of H2O and formation of O2, H+ and electron.
iii) A positive ion which is difficult to get discharged at cathode, leads to the decomposition
of water, with the formation of H2, OH– and absorption of electron.

Let us consider electrolysis of molten salts (e.g. NaCl) and then aqueous solution of salts in an
electric cell shown below.

Molten NaCl (an electrolyte) means free sodium ions (Na+) and chloride ions (Cl–), so it conducts
current with the help of ions. As electric current is passed in the cell, Cl– ions are attracted to
anode (+ve electrode) and Na+ ions to cathode (–ve electrode). Both ions are discharged at
respective electrode as follows:
Anode: 2Cl– → Cl2(g) + 2e–
Cathode: Na+ + e– → Na(s)
Overall Reaction: 2Na+ + 2Cl– → 2Na(s) + Cl2(g)
Let us now consider electrolysis of aqueous solution of salts (e.g., Na2SO4). In aqueous solution,
Na+ ions and SO42– ions are free to move for the conduction of electric current. When electric
current is passed. Na+ ions are attracted to cathode and SO42– ions to anode, but they are not
discharged. The electrode reactions are as follows:

Cathode 2H2O + 2e– → H2 + 2OH– or 2H+ + 2e– → H2


Anode 2H2O → O2 + 4H+ + 4e– or 4OH– → O2 + 2H2O + 4e–
Overall Reaction 2H2O → O2 + 2H2

In this case, H2 is liberated to cathode and O2 at anode.

Electrolysis of Sodium Chloride Solutions: When you electrolyze an aqueous solution of


sodium chloride, NaCl, the possible species involved in half-reactions are Na+, Cl– and H2O. The
possible cathode half-reactions are

Na+(aq) + e– → Na(s) E0 = –2.71V


2H2O(l) + 2e– → H2(g) + 2OH–(aq) E0 = – 0.83V

Under standard conditions, you expect H2O to be reduced in preference to Na+, which agrees
with what you observe. Hydrogen is evolved at the cathode.

[Type text]
[Type text]

2Cl–(aq) → Cl2(g) + 2e– –E0 = –1.36V


2H2O(l) → O2(g) + 4H+(aq) + 4e– –E0 = – 1.23V

Under standard-state conditions, you might expect H2O to be oxidized in preference to Cl–.
However, the potentials are close and over voltages at the electrodes could alter this conclusion.
It is possible nevertheless to give a general statement about the product expected at the anode.
Electrode potentials, as you have seen, depend on concentrations. It turns out that when the
solution is concentrated enough in Cl–, Cl2 is the product; but in dilute solution, O2 is the product.
To see this, you would simply apply the Nernst equation of the Cl–|Cl2 half reaction.

2Cl–(aq) → Cl2(g) + 2e–

Starting with very dilute NaCl solutions, you would find that the oxidation potential of Cl– is
very negative, so H2O is reduced in preference to Cl–. But as you increase the NaCl
concentration, you would find that the oxidation potential of Cl– increases until eventually Cl– is
oxidized in preference to H2O. The product changes from O2 to Cl2.

The half-reactions and cell reaction for the electrolysis of aqueous sodium chloride to chlorine
and hydroxide ion are as follows:

2H2O(l) + 2e– → H2(g) + 2OH–(aq) (cathode)


2Cl (aq) → Cl2(g) + 2e
– –
(anode)
–––––––––––––––––––––––––––––––––––––––––
2H2O(l) + 2Cl–(aq) → H2(g) + Cl2(g) + 2OH–(aq)

Because the electrolysis started with sodium chloride, the cation in the electrolyte solution is Na+.
When you evaporate the electrolyte solution at the cathode, you obtain sodium hydroxide NaOH.

In General

i) Of the two cations, that cation is liberated at the cathode which has higher reduction
potential. Of the two anions, that anion will be discharged at the anode which has lower
reduction potential.

ii) If electrode is active at cathode, metal goes on depositing on cathode and at anode metal
is dissolved.

iii) It must be noted that it is not the SRP of a cation that decides its discharge, but its
reduction potential. The SRP’s should be used only when the concentration of ions are
1M. For concentrations other than 1M, the reduction potentials for each cation at the
concentration is calculated and then the discharge of an ion can be predicted.

iv) The discharge of NO3− , PO34− and SO 24− ions at anode does not commonly take place from
aqueous solution.

[Type text]
[Type text]

Laws of Electrolysis
The laws which govern the deposition of substances (in the form of ions) on electrodes with
passage of electric current are called Faraday’s laws of electrolysis.

1. Faraday’s First Law: The mass of an ion discharged during electrolysis is


proportional to the quantity of electricity passed. Mathematically, W∝ Q

Where W = Mass of ions liberated


Q = Quantity of electricity passed in coulombs
But we know that Q = Current in amperes (c) × times in sec. (t)
∴ W ∝ C × t or W = Z × C × t

Where Z is a constant, known as electrochemical equivalent (ECE) of the ion deposited.

When a current of 1 ampere is passed for 1 second (i.e. Q = 1), then W ∝ Z

Thus electrochemical equivalent may be defined as the mass of the ion deposited by
passing a current of one ampere for one second (i.e. by passing one coulomb of
electricity). Its unit is kg C–1.

1F of charge = Charge on one mole of electron

= NA × e–
= 6.023 × 1023 × 1.602 × 10–19
= 96514.8C ≈ 96500C
= The charge which discharges E g of ion where E is equivalent weight of ion
Thus F = NA × e–
Since 96500C discharges E g of ion
E
∴1C discharge = g of ion =Z
96500
Now substituting the value of Z in the reaction.
ECT
W = ZCT =
96500
W Ct Q Q
or = = =
E 9650 96500 F

2. Faraday’s Second Law: When the same quantity of electricity is passed through
different electrolytes, the masses of different ions liberated at the electrodes are directly
proportional to their chemical equivalents (equivalent weights). Suppose W1 and W2 are
the weights of the elements deposited by passing a certain quantity of electricity through
their salt solutions and E1 and E2 are their respective equivalent weights, then

[Type text]
[Type text]

W1 E1 Z Ct E
= of 1 = 1 ( ∴ W=ZCt)
W2 E 2 Z2Ct E 2
Z E
∴ 1 = 1
Z2 E 2

Thus the electrochemical equivalent (Z) of an element is directly proportional to its


equivalent weight (E), i.e. E ∝ Z or E = FZ

Where F is again a proportionality constant and has been found to be 96540 coulombs. It
is called Faraday. Thus E = 96540 × Z

Therefore, when 96500 (or roughly 96500) coulombs of electricity is passed through an
electrolyte, one gram equivalent of its ions is deposited at the respective electrode. This
quantity of electricity which liberates one gram equivalent of each element is called one
Faraday and is denoted by F.

∴ 1 Faraday = 96500 coulombs

Illustration 12: In a fuel cell, hydrogen and oxygen react to produce electricity. In the process of
hydrogen gas is oxidized at the anode and oxygen at the cathode. If 67.2 litre of H2 at
STP react in 15 minutes, what is the average current produced? If the entire current is
used for electro-deposition of copper from copper (II) solution, how many grams of
copper will be deposited?
Anode reaction: H2 + 2OH– → 2H2O + 2e–
Cathode reaction: O2 + 2H2O + 2e → 4OH–

Solution: From the given reactions, it is obvious that


22.4 litres of H2 gas require 2 Faraday electricity
∴ 67.2 litres of H2 will require = 6 Faraday electricity
Q=C×t
6 × 96500 = C × 15 × 60
6 × 96500
∴C= = 643.3 ampere
15 × 60
Calculation of amount of Cu deposited by 6F
Since 1F deposits = 31.75 × 6g = 190.50g

Some Commercial Cells


The commercial cells are actually portable source of electrical energy. These cells are also known
as Batteries.

Types of Commercial Cells: There are mainly two types of commercial cells.

a) Primary Cells: These cells cannot be longer period of time because they are not
chargeable. Some common examples of primary cells are:

[Type text]
[Type text]

i) Dry cell and


ii) Mercury Cell

i) Dry Cell: It is used in torches, toys and in other electronic devices. Leclanche cell is
the compact form of the dry cell.

In this cell zinc cylinder acts as an anode. The cylinder is filled with the paste of
NH4Cl and little amount of ZnCl2 in the form of paste in water. A graphite rod
surrounded by a black paste of MnO2 and carbon powder acts as a cathode.

The cathodic reaction of the cell is represented as

MnO2 + NH4+ + e– → MnO(OH) + NH3

And the anodic reaction is Zn → Zn2+ + 2e–

It gives voltage of approximately 1.2 to 1.5V

Since, the NH4Cl in slightly aqueous medium is acidic, it will corrode the Zn and the
cell stops its functioning. Due to the corrosion and the consumption of Zn in the
process, this cell cannot be reused.

ii) Mercury Cell: It consists of Zn anode and HgO(s) as a cathode. The electrolyte of
this cell is a paste of KOH and ZnO.

The anodic reaction of the cell: Zn(s) + 2OH– → ZnO(s) + H2O + 2e–
And the cathodic reaction is HgO(s) + H2O + 2e– → Hg(l) + 2OH–

The voltage of the mercury cell is 1.35V. The cell shows constancy in its potential
throughout its life. This cell is used in small electrical circuits e.g., watches, hearing
aids etc.

Secondary Cells: They are rechargeable by passing direct current in them, and, therefore,
they can be used again and again. These cell are also known as accumulators or storage cells.

Most common accumulators are Lead Accumulators and Nickel Cadmium Accumulators.

Lead Accumulators: In lead accumulators three to six voltaic cells are connected in series to get 6
to 12 volt.

The anode of this cell is made of spongy lead and cathode is a grid of lead packed with PbO2.
The electrolyte used in the cell is 38% H2SO4 by weight having a density of 1.30 g/ml. When the
Pb plates are placed for sometimes, a deposit of PbSO4 is formed on them.

The cell (voltaic) is represented as:

Pb | PbSO4 || H2SO4(aq) | PbSO4 | PbSO2 | Pb

[Type text]
[Type text]

Pb(s) + SO42–(aq) PbSO4 + 2e–


2e– + PbO2(s) + 4H+(aq) + SO42–(aq) PbSO4(s) + 2H2O
Pb(s) + PbO2(s) + 2H2SO4(aq) PbSO4(s) + 2H2O (overall reactions)

It is clear from the net cell reaction that H2SO4 is consumed during the operation of the cell and
its density regularly decreases. When the density of the H2SO4 solution falls below 1.2g/ml, the
recharging of the battery is required.

Recharging of the Cell: When D.C. passed directly in this battery, the reactions of its voltaic
cells are reversed and this will slowly increase the density of the H2SO4 solution.

The recharging reaction is represented as

2PbSO4(s) + 2H2O → Pb(s) + PbO2(s) + 2H2SO4

The discharging process of the storage cell is based on the principles of electrochemical cell,
whereas the charging process is based upon the principles of electrolytic cells.

Illustration 13: A completely charged lead storage cell is found to have 1.6 litre of 6 M H2SO4 of 3.5
ampere of current is consumed for 6 hours, what is the concentration of sulphuric acid
and in the cell?

Solution: The overall reaction is


Pb(S) + PbO2(s) + 2H2SO4 → 2PbSO4(s) + 2H2O
3.5 × 6 × 60 × 60
∴ Moles of electron used = = 0.783
96500
This equals moles of H2SO4 used.
∴ Moles of H2SO4, left = 9.6 – 0.783 = 8.817
8.817
∴ Molarity of H2 SO4 left = = 5.51
1.6

Electrolytic Conductance

Specific Conductance
The resistance (R) of a metallic conductor is directly proportional to its length (l) and inversely
proportional to its cross-sectional area (a), i.e.,
l l
R∝ ⇒ R = ρ⋅
a a
1 1 l
⇒ = ⋅ …(1)
ρ R a

[Type text]
[Type text]

Where ρ is a constant depending upon the nature of the material and is called specific resistance
of the material.

The reciprocal of the resistance is called conductance or conductivity and similarly, the
reciprocal of specific resistance is called specific conductance,

Thus, from equation (1)


l
Specific conductance = observed conductance × …(2)
a
When R = 1 cm and a = 1 cm2
Specific conductance = observed conductance

Thus, specific conductance is the conductance of a conductor which is observed when it is 1 cm


in length and 1 sq. cm in cross-sectional area. In other words, it is the conductance of 1 cc of the
conductor.

The unit of resistance is ohm (Ω) so unit of conductance will be ohm–1, mho or Ω–1 and
expressing l in cm and a in cm2, the unit of specific conductance will be ohm–1cm–1,
Ω–1 cm–1 or mho cm–1. In SI system, the units of specific conductance are Sm–1 where S stands
for Siemen.

The equation (2) mentioned above is also applicable for solution of an electrolyte i.e. electrolytic
conductor. The specific conductance of an electrolytic solution is defined as the conductance that
is observed when two electrodes each of 1 sq. cm in cross-sectional are dipped into solution at a
distance of 1 cm apart. In other words, specific conductance is the conductance per c.c. solution
of the electrolyte. It is denoted by the symbol κ (kappa). Sometimes κv or κc is also used, the
subscript v or c standing for dilution or concentration, respectively, signifying that κ is dilution
or concentration dependent.

Equivalent Conductance
Equivalent conductance is the conducting power of all the ions produced by one g-equivalent i.e.
one equivalent of an electrolyte in a given solution. The equivalent conductance may, therefore,
be defined as the conductance which is observed when two sufficiently large electrodes are
dipped into solution at such a distance so as to enclose in between them the entire volume of
solution containing one equivalent of the electrolyte.

It is denoted by the symbol ∧.

Let one equivalent of an electrolyte is dissolved in v c.c. solution. Then all the ions produced by
1 equivalent of the electrolyte will be present in this v c.c. solution. So, the conductance of this v
c.c. solution will be the equivalent conductance of the electrolyte i.e.

∧ = Conductance of v c.c. solution containing one equivalent of the dissolved electrolyte.


= Conductance of 1 c.c solution × v

[Type text]
[Type text]

= κ×v …(3a)
Where v = volume of solution in c.c containing 1 equivalent of the electrolyte

If C be the normality of solution i.e. concentration of electrolytic solution in equivalent/L, then

1000
V=
C
1000 κ
∴∧= …(3b)
C
Unit of ∧: Ohm–1 cm–1 × cm3 i.e. Ohm–1 cm2 or Ω–1cm2

Molar Conductance
The recent trend is to describe electrolytic conductance in terms of molar conductance which is
defined as the conductance of solution due to all the ions produced by one mole of the dissolved
electrolyte in a given solution.

It is denoted by the symbol ∧m


∧m and κ are inter-related as
1000 κ
∧m = κ × v ⇒ ∧m = …(4)
C
Where v = Volume of solution in c.c. containing one mole of the electrolyte and
C = Concentration of solution in mole L–1 i.e. molarity
The above inter-relationship may also be expressed as
κ
∧m =
C
Where C is the concentration of the solution in moles per cubic metre.
Unit of ∧m: Ω–1cm2mol–1
In SI system it is S m2 mol–1

Relation between ∧ and ∧m;


∧m = n∧
M
Where n = n-factor of the electrolyte = total charge carried by either ion =
E
BRAIN TEASER 1:
Find equivalent and molar conductance of 0.01 M solution of K2SO4 whose specific
conductance is 1.26 × 10–3 Ω–1 cm–1.

Variation of Conductance with Dilution


Upon dilution i.e. lowering concentration, specific conductance decreases while equivalent and
molar conductances increase. At infinite or almost zero concentration equivalent conductance
and molar conductance attain their respective limiting values called equivalent conductance at

[Type text]
[Type text]

infinite dilution (∧∞) or zero concentration (∧0) and molar conductance at infinite dilution ∧ ∞m ( )
or zero concentration ( ∧( ) , respectively. The increase of ∧ or ∧m with dilution of a weak
0
m

electrolyte is attributable to increase of degree of dissociation with dilution resulting into more
number of ions in solution. Note that ions are carriers of electricity. The increase of ∧ and ∧m of a
strong electrolyte which remains completely ionised at all dilutions, is attributed to increase in
the ionic mobilities of ions due to decrease in inter-ionic attraction. As dilution approaches
infinity, the degree of dissociation of weak electrolyte approaches unity, the number of ions
becomes maximum and hence ∧ as well as ∧m approach their respective maximum value. In the
case of strong electrolyte, however, the maximum value of ∧ or ∧m is attained due to the
maximum ionic mobilities of the ions since at infinite dilution the dissociation of strong
electrolyte is complete and inter-ionic attraction ceases to exist completely.

The decrease in κ may also be explained in the following way: Upon dilution the number of ions,
in the case of weak electrolyte, increases but volume of solution also increases. The increase of
volume is in greater proportion than the increase of number of ions resulting into decrease in
number of ions per c.c. solution. The specific conductivity being the conductivity of 1 c.c.
solution, should obviously decrease.

The variation of molar conductance of a strong electrolyte with concentration is theoretically


given by Debye-Hückel-Onsager equation:
(
∧ m = ∧ 0m − A + B ∧ 0m C )
Where A and B are the Debye-Hückel constants depending upon nature of the solvent and
temperature and C is the molar concentration of solution.

For aqueous medium at 25°C:


(
∧ m = ∧ 0m − 60.2 + 0.229 ∧ 0m ) C

According to this equation a plot of ∧m vs C should be a straight line having the slope equal to
60.2 + 0.229 ∧ 0m and intercept equal to ∧ 0m . This has been checked in the case of a number of
univalent electrolytes and found to be positive for C ≤ 0.02 M.

At higher concentration, the observed deviation from linearity is attributable to large inter-ionic
attraction.

Determination of Conductance (κ, ∧ and ∧m)

As already mentioned above


l
κ = Observed conductivity ×
a
l
For a given conductivity cell in a given experiment: = constant called cell constant (x). Thus,
a

[Type text]
[Type text]

1
κ = Observed conductance × x = ×x
Observed resistance

The resistance of a solution is determined by Wheatstone bridge method using a meter bridge the
conductivity cell remains dipped in the test solution. The current used is AC.

The specific conductance of 0.1M KCl solution is known. The resistance of 0.1M KCl solution is
first determined experimentally and thereby cell constant is calculated. The KCl solution is
removed from the cell, it is washed with conductivity water and then filled with test solution. The
resistance of the test solution is measured and since cell constant is already known so specific
conductance of the test solution can be calculated.

From specific conductance, we determine ∧ and ∧m using the equation (3) and equation (4)
respectively.

Determination of ∧ 0m or ∧ 0
A plot of ∧m vs C as found experimentally is as shown below graphically.
∧m
0

Strong electrolyte

∧m

Weak electrolyte

The ∧m vs C plot of strong electrolyte being linear it can be extrapolated to zero concentration.
Thus, ∧m values of the solution of the test electrolyte are determined at various concentrations the
concentrations should be as low as good.

∧m values are then plotted against C when a straight line is obtained. This is the extrapolated to
zero concentration. The point where the straight line intersects ∧m axis is ∧ 0m of the strong
electrolyte.

However, the plot in the case of weak electrolyte being non linear, shooting up suddenly at some
low concentration and assuming the shape of a straight line parallel to ∧m axis. Hence
extrapolation in this case is not possible. Thus, ∧0 of a weak electrolyte cannot be determined
experimentally. It can, however, be done with the help of Kohlrausch’s law to be discussed later.

BRAIN TEASER 2:
The specific conductivity of 0.1 N KCl solution at 20°C is 0.0212 ohm–1 cm–1. This solution
was found to offer resistance of 55 ohm. Find cell constant of the conductivity cell. If a
solution of 1.0 M AB, a univalent electrolyte offers a resistance of 25 ohm, using the same
conductivity cell, find its equivalent conductance.

[Type text]
[Type text]

Kohlrausch’s Law of Independent Migration of Ions


Kohlrausch determined ∧0 values of pairs of some strong electrolytes containing same cation say
KF and KCl, NaF and NaCl etc., and found that the difference in ∧0 values in each case remains
the same.
∧ 0m (KCl) − ∧ 0m (KF) = ∧ 0m (NaCl)− ∧ 0m (NaF)

He also determined ∧0 values of pairs of strong electrolytes containing same anion say KF and
NaF, KCl and NaCl etc. and found that the difference in ∧0 values in each case remains the same.

∧ 0m (KF) − ∧ 0m (NaF) = ∧ 0m (KCl) − ∧ 0m (NaCl)

This experimental data led him to formulate the following law called Kohlrausch’s law of
independent migration of ions.

At infinite dilution when dissociation is complete, every ion makes some definite contribution
towards molar conductance of the electrolyte irrespective of the nature of the other ion which
with it is associated and that the molar conductance at infinite dilution for any electrolyte is given
by the sum of the contributions of the two ions. Thus

∧ 0m = λ 0+ + λ 0− …(5)

Where λ 0+ is the contribution of the cation and λ 0− is the contribution of the anion towards the
molar conductance at infinite dilution. These contributions are called molar ionic conductances at
infinite dilution. Thus, λ 0+ is the molar ionic conductance of cation and λ 0− is the molar ionic
conductance of anion, at infinite dilution. The above equation (5) is, however, correct only for
binary electrolyte like NaCl, MgSO4 etc.

For an electrolyte of the type of AxBy, we have


∧ 0m = xλ 0+ + yλ 0−

Application of Kohlrausch’s Law

i) Determination of ∧ 0m of a weak electrolyte: In order to calculate ∧ 0m of a weak


electrolyte say CH3COOH, we determine experimentally ∧ 0m values of the following
three strong electrolytes:
a) A strong electrolyte containing same cation as in the test electrolyte, say HCl
b) A strong electrolyte containing same anion as in the test electrolyte, say CH3COONa
c) A strong electrolyte containing same anion of (a) and cation of (b) i.e. NaCl.
∧ 0m of CH3COOH is then given as:
∧ 0m (CH3COOH) = ∧ 0m (HCl) + ∧ 0m (CH3COONa) – ∧ 0m (NaCl)
Proof:

[Type text]
[Type text]

∧ 0m (HCl) = λ 0H + λ Cl− …I
∧ 0m (CH3COONa) = λ 0CH COO− + λ Na + …II
3

∧ 0m (NaCl) = λ 0Na + + λ 0Cl− …III


Adding equation (I) and equation (II) and subtracting(III) from them:
∧ 0(HCl) + ∧ (CH
0
3COONa )
− ∧ 0( NaCl) = λ (H
0
+ +λ
)
0
(CH COO0 )
= ∧ 0(CH 3COOH)
3

BRAIN TEASER 3:
For the strong electrolytes NaOH, NaCl and BaCl2, the molar ionic conductances at infinite
dilution are 248 × 10–4, 126.5 × 10–4and 280.0 × 10–4 s m2 mol–1, respectively. Calculate
∧ 0m for Ba(OH)2.

ii) α):
Determination of degree of dissociation (α

No.. of molecules ionised ∧


α= = 0m
total number of molecules dissolved ∧ m

iii) Determination of solubility of sparingly soluble salt: The specific conductivity


of a saturated solution of the test electrolyte (sparingly soluble) made in conductivity
water is determined by the method as described above. From this the specific conductivity
of conductivity water is deducted. The molar conductance of the saturated solution is
taken to be equal to ∧ 0m as the saturated solution of a sparingly soluble salt is extremely
dilute. Hence from equation (4).
1000 κ
∧ 0m = ,
C
Where C is the molarity of solution and hence the solubility.

BRAIN TEASER 4:
The specific conductance of a saturated solution of AgCl at 25°C after subtracting the
specific conductance of water is 2.28 × 10–4 S m–1. Calculate the solubility of AgCl in gram
per dm3 at this temperature. The molar ionic conductances of Ag+ and Cl– ions are
73.3 × 10–4 and 65.0 × 10–4 Sm2 mol–1, (Ag: 108 and Cl: 35.5)

iv) Determination of ionic product of water: From Kohlrausch’s law, we determine


∧ 0m of H2O where ∧ 0m is the molar conductance of water at infinite dilution when one
mole of water is completely ionised to give one mole of H+ and one mole of OH– ions i.e.

∧ 0m (H2O) = λ 0H+ + λ 0OH−


Again using the following
κ ×1000
∧m = , where C = molar concentration i.e. mol L–1 or mol dm–3
C

[Type text]
[Type text]

κ
⇒ ∧m = , where C = concentration in mol m–3
C
Assuming that ∧ m differs very little from ∧ 0m
κ κ
∧ 0m = ⇒C= 0
C ∧m
Specific conductance (κ) of pure water is determined experimentally. Thereafter, molar
concentration of dissociated water is determined using the above equation.

Kw is then calculated as: Kw = C2

Answers to Brain Teaser

Brain Teaser 1: 63 Ω–1 cm2 equiv–1


and 1.26 × 102 Ω–1 cm2 mol–1
Brain Teaser 2: x = 1.166 cm–1
∧ = 66.66 ohm–1 cm2
Brain Teaser 3: 523.2 × 10–4 S m2 mol–1
Brain Teaser 4: 1.80 × 10–3 g dm–3

Solved Problems
Objective

Problem 1: In the electrolysis of CuCl2 solution using Cu electrodes, the weight of Cu anode
increased by 2 gram at cathode. In the anode.
(a) 0.2 mole of Cu2+ will go into solution.
(b) 560 ml O2 liberate
(c) No loss in weight
(d) 2 gram of copper goes into solution as Cu2+

Solution: (d) During electrolysis of CuSO4 solution using Cu electrodes, the cell reaction is
Anode: Cu(s) → Cu++ + 2e–
Cathode: Cu++ (aq) + 2e– → Cu(s)
The loss in weight of anode is equal to gain in weight of cathode.

Problem 2: During electrolysis of CuSO4 using Pt-electrodes, the pH of solution


(a) increases (b) decreases
(c) remains unchanged (d) cannot be predicted

Solution: (b) In presence of inert electrode, the cell reaction is


Anode 2H2O → 4H+ + 4e– + O2
Cathode: Cu++(aq) + 2e– → Cu(s)] × 2
–––––––––––––––––––––––––––––––––––––

[Type text]
[Type text]

Net cell reaction 2Cu++ aq. + 2H2O → 4H+ + 2Cu(s) + O2↑


Due to increase in [H+], pH decreases.

Problem 3: A gas X at one atm is bubbled through a solution containing a mixture of 1 MY
and 1 M Z– at 25°C. If the reduction potential of Z > Y > X, then
(a) Y will oxidize X and not Z (b) Y will oxidize Z not X
(c) Y will oxidize both X and Z (d) Y will reduce both X and Z

Solution: (a) The tendency to gain electron is in the order


Z>Y>X
∴ Y + e → Y–
X → X+ + e

Problem 4: The standard reduction electrode potential values of elements A, B, C are + 0.68,
–2.50, and – 0.50 V respectively. The order of their reducing power is
(a) A >B > C (b) A > C > B
(c) C > B > A (d) B > C > A

Solution: (d) More is the reduction potential, more is the power to get itself reduced or
lesser is reducing power or greater is oxidizing power.

Problem 5: The value of equilibrium constant for feasible cell reaction is


(a) < 1 (b) zero
(c) = 1 (d) > 1

0.059
Solution: (d) E0 = logK, if E0 is +ve, thus K > 1
n

Problem 6: How much will potential of a hydrogen electrode change when its solution
initially at pH = 0 is neutralized to pH = 7
(a) increase by 0.0591 V (b) decrease by 0.0591 V
(c) increase by 0.413 V (d) decrease by 0.413 V

1
Solution: (d) H++ e– H2(g),
2

0.0591
[E = E0 – log Q]
n
P1
0.0591 H 22 0.591 1
= 0.0 – log +
= log −7
1 [H ] 1 10
= – 0.0591 × 7 × log10 = – 0.413 V

Problem 7: Given that


E 0Fe2+ / Fe = – 0.44 V, and

[Type text]
[Type text]

E 0Fe3+ / Fe2+ = 0.77V


What is E 0Fe3+ / Fe = ?
(a) – 0.11V (b) 0.11V
(c) – 0.04 V (d) 0.04 V

n1E10 + n 2 E 02 2 × (−0.44V) + 1× 0.77V


Solution: (c) E =0
3 = = –0.04V
n3 3

Problem 8: Following are some standard reduction potential values for the given half cell:
i) A++ + 2e– A E0 = 1.27 V
ii) B+ + e– B E0 = – 0.7 V
iii) C++ + 2e– C E0 = – 0.54 V
+ –
iv) D + e D E 0 = 1.05 V
The combination of which two half cells will give galvanic cell having maximum
possible emf.
(a) (i) and (ii) (b) (I) and (iv)
(c) (ii) and (iii) (d) (iii) and (iv)

Solution: (a) Since all the values are standard reduction potential and so the two half cells
having maximum and minimum reduction potential values will give a cell of
maximum possible emf.
E 0cell = E 0RP(cathode) − E 0RP(anode)

Problem 9: By how much would the oxidising power of the MnO −4 / Mn 2 + couple change if the
H+ ions concentration is decreased 100 times?
(a) increases by 189 mV (b) decreases by 189 mV
(c) will increase by 19 mV (d) will decrease by 19 mV

Solution: (b) MnO −4 + 5e– + 8H+ → Mn2+ + 4H2O


According to Nernst equation,
0.059  [Mn 2 + ] 
o
Ered = Ered – log  − + 8
Let [H+]initial = X
5  [MnO 4 ][H ] 
−0.0591  [Mn 2 + ]  X X
Ered(initial) = E ored log  − 8
[H+]final = = 2
5  [MnO 4 [X]  100 10
−0.0591 [Mn 2+ ] × 1016
Ered(final) = E ored log
5 [MnO 4− ] × [X]8
−0.0591
Ered(final) – Ered(initial) = log 1016 = – 0.1891 V
5

[Type text]
[Type text]

This Ered decreases by 0.189 V. The tendency of the half cell to get reduced is
its oxidising power. Hence the oxidising power decreases by 0.189V

Problem 10: A solution of sodium sulphate in water is electrolysed using inert electrodes. The
products at cathode and anode are respectively.
(a) H2,O2 (b) O2,H2
(c) O2,Na (d) O2,SO2

Solution: (a) Na+ ions are not reduced at cathode and SO24− ions are not oxidized at anode
Cathode: 2H2(l) → O2 (g) + 4H+ (aq) + 4e–
Anode: 2H2O(l) + 2e– → H2(g) + 2OH– (aq)
2+
Problem 11: The standard oxidation potentials of Cu/Cu and Cu+/Cu2+ are – 0.4V and
– 0.16V respectively. The standard electrode potential of Cu+/Cu would be
(a) 0.18V (b) 0.52V
(c) 0.82V (d) 0.49V

Solution (b) Reactions


Cu2+ + 2e– → Cu ∆G0 = – nFE0
Cu+ → Cu2+ + e– ∆GI0 = –2F × 0.34 = 0.68F
∆GII0 = F × 0.16 = 0.16F
Adding, we get
Cu+ + e– → Cu ∆G0III = ∆G0I + ∆G0II
= 0.52F = – FE0
∴E0 = 0.52V

Problem 12: Acidified water is electrolysed using an inert electrode. The volume of gases
liberated at STP is 0.168L. The quantity of charge passed through the acidified
water would be
(a) 96,500C (b) 9,650C
(c) 965C (d) 168C

Solution: (c) 2H2O → 2H2(g) + O2(g)


2x x
∴ 3x = 0.168
∴ x = 0.056L
VH2 = 2x = 0.112L, VO2 = x = 0.056L
11.2L of H2 at STP ≡ 1F
0.112L of H2 at STP ≡ 0.01F
0.056L of O2 at STP = 0.01F
∴ The amount of electricity passed = 0.01F = 965C
Problem 13: The useful work done during the reaction

[Type text]
[Type text]

1
Ag(s) + Cl2(g) → AgCl(s)
2
Would be
(a) 110kJ mol–1 (b) 220 kJ mol—1
(c) 55kJ mol–1 (d) 100 kJ mol–1
Given E 0Cl / Cl− = 1.36V, E 0AgCl/Ag/Cl- = 0.220V , PCl2 = 1 atm and T = 298K
2

Solution: (a) For the cell reaction


1
Ag(s) + Cl2(g) → AgCl(s)
2
E0 = – 1.14V
0.0592
or E = E0 – log PCl1/22
1
Under standard conditions, PCl2 = 0
∴ log PCl1/22 = 0
∴Useful work = – Wmax = – nFE
= (–1) × (–1.14) × 96500 × 10–3 kJ = 110 kJ mol–1

Problem 14: A current of 0.250 A is passed through 400 ml of a 2.0 M solution of NaCl for 35
minutes. What will be the pH of the solution after the current is turned off?
(a) 12.98 (b) 12.13
(c) 10.48 (d) 9.24

Solution (b) After electrolysis aqueous NaCl is converted into aqueous NaOH.
0.250 × 35 × 60
The quantity of electricity passed = F
96500
= 5.44 × 10–3F
The number of equivalents of OH– ion formed = 5.44 × 10–3
5.44 × 10−3
∴ Molarity of NaOH = = 1.36 × 10−2
0.4L
∴ pOH = – log (1.36 × 10–2) = 1.87
∴ pH = 12.13

Problem 15: How much charge should be supplied to a cell for the electrolytic production of
245 gm NaClO4 from NaClO3 if the anode efficiency for the required reaction is
60%?
(a) 6.43 × 105C (b) 6.67F
(c) 6.43 × 106 (d) 66.67F

Solution: (a) ClO4– + 2H+ + 2e– → ClO3– + H2O


245
Number of equivalents of NaClO4 = = 4 ≡ 4F
61.25
4 × 100
= 6.67F = 6.43 × 10 C
5
No. of Faradays =
60

[Type text]
[Type text]

Problem 16: The volume of gases liberated at STP when a charge of 2F is passed through
aqueous solution of sodium phosphate, is
(a) 11.2L (b) 44.8L
(c) 33.6L (d) 22.4L
Solution: (c) At anode 2H2O → O2 + 4H+ + 4e–
At cathode 2H2O + 2e– → 2OH– + H2
After passage of 2F, one mole of H2 and 1/2 mole of O2 would be produced.
The total volume is 33.6L.
–1
Problem 17: Specific conductance of 0.01 M KCl solution is x ohm cm–1. When conductivity
cell is filled with 0.01 M KCl the conductance observed is y ohm–1. When the
same cell is filled with 0.01 M H2SO4, the observed conductance was Z ohm–1 cm–
1
. Hence specific conductance of 0.01 M H2SO4 is:
z
(a) xz (b)
xy
xz xy
(c) (d)
y z

Specific conductance x
Solution: (c) Cell constant = = cm −1
Observed conductance y
Specific conductance of 0.01 M H2SO4
x
= Observed conductance × Cell constant = z × ohm–1 cm–1
y

Problem 18: Among the cation H+, Li+, Na+ and K+, the one with highest ionic mobility and
another with lowest ionic mobility respectively are:
(a) [H+, K+] (b) [K+, H+]
+ +
(c) [LI , Na ] (d) [H+, Li+]

Solution: (d) H+ ion has the maximum ionic mobility which is explainable by Grothus
mechanics, Li+ having the maximum charge density, is the most hydrated ion
among the lot and hence the lowest ionic mobility.
Problem 19: The specific conductance has the unit:
(a) ohm–1 cm–1 (b) ohm.cm
(c) ohm cm–1 (d) ohm–1.cm

l
Solution: (a) Specific conductance = Observed conductance ×
a
cm
= ohm −1 × 2
= ohm–1⋅cm–1.
cm
Problem 20: The specific conductivity of solution depends upon:

[Type text]
[Type text]

(a) Number of ions as well as mobility of ions


(b) Number of ions per c.c solution
(c) Number of ions per cc as well as mobilities of ions
(d) Mobilities of ions only
Solution: (c) Specific conductance is the conductance per c.c. solution
Subjective

Problem 1: The emf of the following cell is – 0.46V


Pt(H2) HSO3− (0.4M), SO32− (6.4 × 10–3M) || Zn2+ (0.3M) | Zn
If E 0Zn 2+ / Zn = – 0.76V, calculate pKa of HSO3− i.e. for the equilibrium
HSO3− H + + SO32−

Solution: Half cell Reaction E0


L.H.S. H2 → 2H+ + 2e– E 0ox = 0.00V
R.H.S. Zn2+ + 2e– → Zn E 0cell = 0.76V
––––––––––––––––––––––––
Net Zn2+ + H2 → Zn + 2H+ E 0cell = 0.76V
––––––––––––––––––––––––
[H + ]2 0.0591
K= = log K
[Zn 2 + ] n
0.0591
E cell = E 0cell − log K
n
0.0591 [H + ]2
−0.46 = −0.76 − log
2 (0.3)
[H+] = 4.60 × 10–6M
HSO3− H+ + SO32 −
0.4M 4.60 × 10–6M 6.4 × 10–3M
[H + ][SO32 − ] 4.6 × 10−6 × 6.4 × 10−3
K= = = 7.36 × 10−8
[HSO3− ] 0.4
pKa = 7.13

Problem 2: The E0 for Ag+/Ag and Cr2O72– in acid medium to Cr+3 are 0.8 V and 1.33V. Find
the value of E and ∆G for the conversion of Cr2O7 to Cr+3 in acid when Ag gets
oxidised to Ag+. The concentrations of dichromate ion, Cr(+3) and hydrogen ion
are.
2M, 0.5 M and 0.25 M, Ag+ = 1 M

Solution: 6Ag → 6Ag+ + 6e E0 = – 0.8 V ...(1)


Cr2O72– + 14H+ + 6e → 2Cr+ + 7H2O E0 = 1.33 …(2)
Adding the both we get,

[Type text]
[Type text]

Cr2O72– + 6Ag + 14H+→ 2Cr+3 + 6Ag+ + 7H2O E0 = 0.53 V


0 2.303RT [Cr +3 ][Ag + ]6
E=E – log n=6
nF [Cr2 O72 − ][Ag]6 [H + ]14
0.059 (0.5)2 × (1)6
= 0.53 – × log
6 2 × (0.25)14
= 0.53 – 0.0098 × 7.525 = 0.53 – 0.073 = 0.457 V
∆G = – nFE = – 6 × 96500 × 0.457 = – 264603 J

Problem 3: Calculate the solubility product and solubility of AgCl in the following cell which
has an emf of 0.455 volts at 25°C.
Ag/AgCl in 0.1 M KCl | 0.1 M AgNO3 | Ag
a1 = ? a2

Solution: Ag → Aga+1 + e–
Ag a+2 + e– → Ag
–––––––––––––––
Ag a+2 → Ag a+1
0.0591 a
E= − log 1
1 a2
a 0.1
= 0.0591 log 2 = 0.059 log = 0.455
a1 a1
0.1 0.455
log = = 7.71
a1 0.059
0.1
= anti log 7.71 = 5.129 × 107
a1
0.1
a1 = [Ag + ] = = 1.95 × 10–9
5.129 × 10 7

[Cl–] = 10–1
Solubility product of AgCl = [Ag+] [Cl–]
= 1.95 × 10–9 × 10–1 = 1.95 × 10–10
Solubility of AgCl = [Ag+] = [Cl–] = 1.95 × 10−10
= 1.396 × 10–5 moles/litre

Problem 4: For the galvanic cell


Ag | AgCl(s) KCl (0.2M) || KBr (0.001 M) AgBr(s) | Ag
Calculate the emf generated and assign correct polarity to each electrode for a
spontaneous process after taking into account the cell reaction at 25°C
[Ksp(AgCl) = 2.8 × 10–10; Ksp (AgBr) = 3.3 × 10–13]

Solution: The given cell may be written as under

[Type text]
[Type text]

Ag|Ag+(C1)Cl– (0.2M) || Ag+ (C2), Br– (0.001 M) | Ag


The above cell in a concentration cell with the cell reaction
[Ag+]RHS → [Ag+]LHS
[Ag + ]LHS
and cell potential = – 0.059 log
[Ag + ]RHS
[K sp AgCl] /[Cl− ]
Ecell = – 0.0591 log
[K sp AgBr] /[Br − ]
2.8 × 10−10 0.001  14 × 10−9 
= – 0.0591 log × = – 0.0591log  33 × 10−10 
0.2 3.3 × 10−13  
= – 0.0591 log 4.242 = – 0.0591 × 0.6276 = – 0.03709 volts
Since cell potential is negative, the cell reaction will be spontaneous in the reverse
direction i.e. AgCl + Br– → AgBr + Cl–
with Cl– | AgCl | Ag serving as cathode (+ve terminal) and Br– | AgBr | Ag as
anode ( –ve terminal). The spontaneity is due to the fact that AgBr is less soluble
than AgCl.

Problem 5: The standard reduction potential for the half cell.


NO3–(aq) + 2H+(aq) + e– → NO2(g) + H2O(l) is 0.78 V
a) Calculate the reduction potential in 8M H+
b) What will be the reduction potential of the half-cell in a neutral solution?
Assume all other species to be at unit concentration

Solution: For the half cell reaction


NO3–(aq) + 2H+(aq) + e– → NO2(g) + H2O(l)
0.059 [Pr oducts]
The Nernst equation is E = E0 – log
n [Re ac tan ts]
0.059 1
= 0.78 – log 2 = 0.78 + 0.059 log 8
2 (8)
= 0.833V
Substituting this value for case (b)
0.059 1
E = 0.78 – log −7 2
2 (10 )
= 0.78 – 0.059 × 7 = 0.367V

Problem 6: Calculate EMF of the cell


M
Pt, H2 (0.1 atm) | solution (pH = 4) || KCl solution (saturated with AgCl) | Ag
100
and equilibrium constant of the cell reaction at 25°C. Given EoAg + ,Ag = 0.80 V and
Ksp (AgCl) = 4.36 × 10–11 at 25°C.

[Type text]
[Type text]

1
Solution: Cell reaction: H 2 + Ag+ → H+ + Ag(s)
2
+
[H ] 10−4
Q= +
= −9
= 2.29 × 104.5
1/ 2
[H 2 ] [Ag ] (0.1) (4.36 × 10 )
1/ 2

(Q pH = 4 ∴ [H+] = 10–4 M. [H2] = PH2 = 0.1 atm, and


4.36 × 10−11
K sp
Ksp = [Ag ] [Cl ], ∴ [Ag ] =
+ –

= +
−2
= 4.36 × 10–9 M)
[Cl ] 10
o
Ecell = E Cell – 0.059 Q

(
= E oAg + ,Ag − E oH + ,1 H
2 2
) – 0.059 log Q
= (0.80 – 0) – 0.059 log (2.29 × 104.5 )
= 0.80 – 0.059 (log 2.29 + 4.5)
= 0.80 – 0.059 (0.3598 + 4.5) =0.514
Alternate method:
E Ag + ,Ag = E oAg + ,Ag + 0.059 log [Ag+]
= 0.80 + 0.059 log (4.36 × 10–9)
= 0.80 + 0.059 (0.6493 – 9)
= 0.80 + 0.0377 – 0.531 = 0.3067
1/ 2
PH 2
E H+ ,1 H = E + 1 − 0.059 log
2 2 H + H2
2 [H + ]
10−1/ 2
= 0 – 0.059 log
10−4
= – 0.059 × 3.5 = – 0.2065
Ecell = E Ag + ,Ag − E o + 1
H , H2
2
= 0.3067 + 0.2065 = 0.5132 V
E = Eo – 0.059 log Q
At equilibrium: E = 0 and Q = K = equilibrium constant
So, Eo = 0.059 log K
0.80 = 0.059 log K
K = 3.622 × 1013

Problem 7: Construct the cell corresponding to the reaction:


3Cr2+ (1M) → 2Cr3+ (1M) + Cr(s)
and predict if the reaction is spontaneous. Also calculate the following
i) ∆H and ∆S of the reaction at 25°C.
ii) ∆G of the reaction at 25°C when Cr2+ = 0.5 M and Cr3+ = 0.25 M
Given EoCr 3 + ,Cr = 0.5 V, EoCr 3 + ,Cr 2 + = – 0.41 V
∆G° of the reaction at 35°C = – 270.50 kJ

[Type text]
[Type text]

Solution: The cell corresponding to the given reaction is as follows.


Pt |Cr2+(1M), Cr3+(1M) || Cr2+ (1M) | Cr(s)
I Cr3+ + 3e → Cr(s), ∆GIo = −3FEoCr 3 + ,Cr
II Cr3+ + e → Cr2+ , ∆GIIO = −FEoCr 3 + ,Cr 2 +
Eq. I – Eq. II: Cr2+ + 2e → Cr(s), ∆G° = – 3 × 0.5 F – 0.41 F
= – 1.91 F
∆G° = – 2 FE oCr 2+ ,Cr = – 1.91 F
1.91
∴ EoCr 2 + ,Cr = = 0.955 V
2
E oCell = E Cr
o
2+
,Cr
– E oCr3+ ,Cr 2+ = 0.955 + 0.41 = 1.365 V
Eocell is +ve so ∆G° is – Ve and hence the given reaction is spontaneous

i) ∆G° = – nF Eocell = – 2× 96500 × 1.365 J


= – 263.44 kJ
From Gibb's – Helmholtz equation
 ∂ (∆G) 
∆G = ∆H + T 
 ∂T  P
(−270.50 + 263.44)
– 263.44 = ∆H + 298
10
= ∆H – 298 × 0.706
∴ ∆H = - 53.05 kJ
∆H − ∆G −53.05 + 263.44
∆S = =
T 298
= 0.706 kJ K–1 = 706 kJ K–1

[Cr +3 ]2 (0.25) 2
ii) ∆G = ∆G° + RT ln Q, Q = = = 0.5
[Cr 2+ ]3 (0.5)3
= – 263.44 + 8.314 × 10–3 × 298 × 2.303 log 0.5
= – 263.44 – 5.705 × .30103 = – 263.44 – 1.717 = – 264.717 kJ

Problem 8: Consider the cell: Pb | PbSO4 | Na2SO4⋅10H2O


The temperature coefficient of the emf of above cell is 0.000174V deg–1 and heat
of reaction is – 176.146 kJ. Calculate emf of the cell.

Solution: The cell reaction is


At anode Pb(s) + SO42– → PbSO4(s) + 2e–
At cathode Hg2SO4(s) + 2e– → 2Hg(l) + SO42–
––––––––––––––––––––––––––––––
Pb(s) + Hg2SO4(s) → PbSO4(s) + 2Hg(l)
i.e. the number of electrons involved are two, substituting the values of known
terms in equation (3)

[Type text]
[Type text]

– 176146 J = – (2) (96500 J V–1) E0 + (298 K ) (96500 J V–1) (2) (0.00174 VK–1)
E0= 0.9645V
–36.4
Problem 9: The solubility product of Fe(OH)2 at 25°C is 10 mol4 dm–12 and E0 (Fe3+/Fe) =
– 0.036 V. Calculate the standard emf of the reaction Fe(OH)3(s) → Fe3+ + 3OH–.

Solution: The cell can be formed as Fe|Fe3+; OH– | Fe(OH)3(s) | Fe


The electrode reactions can be written as
Fe(s) → Fe3+ + 3e–
Fe(OH)3(s) + 3e– → Fe(s) + 3OH–
Overall reaction is i.e., by adding
i.e. E 0 = E 0OH− / Fe(OH) / Fe − E 0Fe2+ / Fe
3

E 0
OH − / Fe(OH)3 / Fe
can be calculated if we know E0 and E0 can be calculated as follows
0.0591 0.0591× (−36.4)
E0 = log10 −36.4 = = 0.75V
0.3 3
E 0OH− / Fe(OH) = −0.75 − 0.036 = −0.786V
3 / Fe

Problem 10: For the following cell


Ag(s) | Ag+ (saturated AgI(aq) || Ag+ (0.10 M) | Ag(s)
Ecell = 0.417. Calculate Ksp of AgI
E 0Ag + / Ag = 0.80V

Solution AgI(s) Ag+(aq) + I–(aq)


Ksp = [Ag+] [I–]
Since [Ag+] = [I–]
∴ Ksp= [Ag+]2
If we manage to calculate [Ag+] in saturated in L.H.S. oxidation half cell (anode),
then Ksp can be calculated.
Oxidation Ag(s) → Ag+ [saturated AgI(aq)] + e–
Reduction Ag+ (0.10 M) + e–→ Ag(s) E 0red = 0.80V
–––––––––––––––––––––––––––––––––––––––––––––––––––––––––––
Net Ag+ (0.10 M) → Ag+ (saturated AgI)(aq) E 0cell = 0.00V
xM
 x 
K = 
 0.1 
Ecell = 0.417V (given)
0.0591
Using Nernst equation E cell = E 0cell − log K
n

[Type text]
[Type text]

0.0591  x 
0.417 = 0.00 − log  
1  0.1 
 x  0.417
log  = = −7.0558
 0.1  0.0591
x = [Ag+] in saturated AgI = 8.79 × 10–9
Ksp = [Ag+]2 = (8.79 × 10–9) = 7.73 × 10–17

Problem 11: The standard electrode potential are for the following reactions:
Fe(s) → Fe2+(aq) + 2e– E0 = 0.48V
Cr3+ + e– → Cr2+(aq) E0 = – 0.41V
If the excess of Fe(s) is added to a solution in which [Cr3+] = 1 M, what will be
[Fe2+] when equilibrium is established at 298K?
Fe(s) + 2Cr3+(aq) Fe2+(aq) + 2Cr3+ (aq)

Solution: Fe(s) + 2Cr3+(aq) Fe2+(aq) + 2Cr3+ (aq)


Initial conc. 1M 0 0
Change –2x +x +2x
Equilibrium (1–2x)M xM 2 ×M
[Fe2+ ][Cr 2+ ]2
K=
[Cr 3+ ]2
x(2x) 2 4x 3
K= = x < 0.5
[Cr 3+ ]2 (1 − 2x) 2
Fe(s) →Fe2+(aq) + 2e– E0 = 0.48V
2Cr3+(aq) + 2e– → 2Cr2+(aq) E0 = – 0.41V
––––––––––––––––––––––––––––––––––––––––––––––
2Cr3+(aq) + Fe(s) → 2Cr2+(aq) + Fe2+(aq) E0cell = 0.07V
When equilibrium is attained Ecell = 0
0.0691
By Nernst equation E cell = E 0cell − log K
2
0.0591
E 0cell = log K
2
0.07 × 2
log K = = 2.3689
0.0591
4x 3
K= = 23.81
(1 − 2x)2
It will be tedious and time consuming to calculate x from cubic equation.
Also x < 0.5 and x can’t be neglected since value of K is high.
The best way is to select range of value of x.
Start with values of x < 0.5

[Type text]
[Type text]

Problem 12: CuSO4 solution is electrolyzed. Predict the electrode reactions and the net
electrolysis reaction when the anode is made of (a) copper and (b) platinum. Also
calculate E0cell of each case.

Solution: We refer E0 values of different reactions form E.C.S.


CuSO4(aq) Cu2+(aq) + SO42–(aq)
Since Cu2+ + 2e– → Cu(s) E0 = 0.34V
Hence in both cases the reduction of Cu2+(aq) to Cu(s) seems quite feasible.
a) At the anode, Cu(s) can be oxidized to Cu2+(aq)
Oxidation Cu(s) → Cu2+(aq) + 2e– E0 = – 0.34V
anode
Reduction Cu2+(aq) + 2e– → Cu(s) E0 = +0.34V
Cathode
The net electrolysis reaction is simply
Cu(s) [anode] → Cu(s) [cathode] E0cell = 0.00V
Only a slight voltage

Problem 13: It is desired to constructed the following voltaic cell to have Ecel = 0.0860V. What
[Cl–] must be present in the cathode half cell to achieve this result?
Ag(s) | Ag+ (satd. AgI(aq)) || Ag+ (satd. AgCl, xMCl–) Ag+(s)
Ksp[AgCl = 1.8 × 10–10, AgI = 8.5 × 10–17]; E 0Ag + / Ag = 0.80V

Solution: At L.H.S. half cell Ag(s) → AgA+ (aq) + e– E 0Ag+ / Ag = −0.80V


At R.HS. half cell AgC+(aq) → AG(s) E 0Ag + / Ag = 0.80V
R.H.S.
––––––––––––––––––––––
Agc+ (aq) → AgA+ (aq) E0cell = 0.00V
Ag+ (L.H.S.) is from Ag( (satd. Aq)
AgI(s) Ag+(aq) + I–(aq)
Ksp = [Ag+] [I–] = [Ag+]2
[Ag+]AgI = [Ag+]A = K sp

= 8.5 × 10−17 = 0.9220 × 10−8 M


Ag+ (R.H.S.) is from AgCl in presence of [Cl–] = xM
AgCl(s) Ag+(aq) + Cl–(aq)
Ksp= [Ag+] [Cl–]
K 1.8 × 10−8
[Ag + ]C = sp− =
[Cl ] x
[Cl–]

[Type text]
[Type text]

0.0591 [Ag + ]A
E cell = E 0cell = log
1 [Ag + ]C
0.9220x × 10 −8
0.0860 = – 0.0591 log
1.8 × 10−10
0.0860
− = log 51.22x
0.0591
– 1.4562 = log 51.22 + logx = 1.7095 + logx
logx = – 3.1647
x = 7 × 10–4M
[Cl–] = 7 × 10–4M

Problem 14: The emf of the cell is 0.788V


Ag | AgI, 0.05 M KI || 0.05 M AgNO3| Ag
Calculate the solubility product of AgI. E 0Ag + / Ag = 0.80V

Solution: KI is strong electrolyte, hence


[I–]L.H.S. = 0.05 M
AgI(s) is sparingly soluble. If we manage to calculate Ag+ (Ag) in L.H.S.,. half
cell, Ksp can be calculated.
at L.H.S. half cell Ag(s) → Ag+ (xM) + e– E 0ox = −0.80V
at R.H.S. half cell Ag+ (0.05) + e– → Ag(s) E 0red = 0.080V
R.H.S.
––––––––––––––––––––––
Net Ag+ (0.05) → Ag+ (xM) E 0cell = 0.00V
––––––––––––––––––––––
[Ag + ]L.H.S. x
K= +
=
[Ag ]R.H.S. 0.05
0.0591
E cell = E 0cell − log K
n
0.0591  x 
0.788 = 0 − log  
1  0.05 
 x 
log   = −13.3333 = 14.6667 l
 0.05 
x
= 4.6416 × 10−14 ; x=2.231× 10-15 x
0.05
[Ag + ]L.H.S. = 2.31× 10−15 M
[I–]L.H.S. = 0.05M
Ksp = [Ag+] [I–] = 2.31 × 10–15 × 0.05 = 1.16 × 10–16

[Type text]
[Type text]

Problem 15: The emf of the following cell 0.265V at 25°C and 0.2595V at 35°C. Calculate
heat of reaction taking place at 25°C. Pt (H2) | HCl(aq) || AgCl | Ag

Solution: L.H.S. half cell H2(g) → 2H+(aq) + 2Cl–(aq) + 2e–


R.H.S. half cell 2AgCl(s) + 2e– → 2Ag+(aq) + 2Cl–(aq)
Net H2(g) + 2AgCl(s) → 2H+(aq) + Cl–(aq) + 2Ag+(aq)
  dE  
∆H (heat of reaction) = nF T   − E
  dt  
dE 0.2595 − 0.265
= = 5.5 ×10 −4
dT 308 − 298
n = 2, F = 96500C, E = 0.265 V at 298K
= – 82777.7J = – 82.8kJ

[Type text]
[Type text]

kvpy

o o
1. A saturated solution of BaSO4 is heated from 25 C to 35 C and the conductance of the solution and
the
solubility of BaSO4 are measured. It is found that : (2008)
(A) Both conductance and solubility increase
(B) Both conductance and solubility decrease
(C) Conductance increases but solubility decreases
(D) Conductance decreases but solubility increases

2. The cell potential of an electrochemical cell with the cell reaction


+ 2+
Zn + 2Ag (0.0001M) Zn (0.1 M) + 2Ag, is (2008)
(A) 1.25 V (B) 1.35 V (C) 1.45 V (D) 1.55 V
Given that the standard cell potential is 1.56 V

3. The order of reactivity of K, Mg, Au and Zn with water is (2013)


(A) K > Zn > Mg > Au (B) K > Mg > Zn > Au (C) K > Au > Mg > Zn (D) Au > Zn > K
> Mg

Answer Key:

1:A 2:B 3:B

[Type text]
[Type text]

Assignments
SECTION – I Single Choice Questions

1. The density of Cu is 8.94 g cm –3 . The quantity of electricity needed to plate an area


10 cm ×10 cm to a thickness of 10 –2 cm using CuSO4 solution is
(a) 13 586 C (b) 27 172 C
(c) 40 758 C (d) 20 348 C.

2. In an electrolysis of an aqueous solution of sulphate, 2.4 L of oxygen at STP was


liberated at anode. The volume of hydrogen at STP liberated at cathode would be
(a) 1.2 L (b) 2.4 L
(c) 2.6 L (d) 4.8 L.

3. In the electrolysis of CuCl2 solution using copper electrode, if 2.5 g of Cu is deposited at


cathode, then at anode
(a) 890 mL of Cl2 at STP is liberated (b) 445 mL of O2 at STP is liberated
(c) 2.5 g of copper is deposited (d) a decrease of 2.5 g of mass take place

4. The reaction occurring at anode when the electrolysis of CuSO4 is done using Pt
electrode, is
(a) Cu → Cu 2 + + 2e – 4 + 2H 2 O → H 2SO 4 + O 2 + 4e
(b) 2SO 2– –

(c) 2H 2 O → O 2 + 4H + + 4e – (d) 2Cu → Cu 22+ + 2e –

5. Which of the following is correct?


(a) Cathode is positive terminal in an electrolytic cell
(b) Cathode is negative terminal in a galvanic cell
(c) Reduction occurs at cathode in either of cells
(d) Oxidation occurs at cathode in either of cells.

6. Which of the following statements is correct?


(a) Oxidation occurs at anode in both galvanic and electrolytic cells
(b) Reduction occurs at anode in both galvanic and electrolytic cells
(c) Reduction occurs at anode in electrolytic cell whereas oxidation occurs at cathode in
a galvanic cell
(d) Oxidation occurs at anode in electrolytic cell whereas reduction occurs a anode in a
galvanic cell.

[Type text]
[Type text]

7. By how much will the potential of half-cell Cu 2+ | Cu Change if the solution is diluted to
10 times at 298 K?
(a) Increases by 59 mV (b) Decreases by 59 mV
(c) Increases by 29.5 mV (d) Decreases by 29.5 mV.

8. Given that E 0 values of Ag + | Ag, K + | K, Mg 2 + | Mg and Cr 3+ |Cr are 0.80 V, –2.93 V,


–2.37 V, and –0.74 V, respectively. Which of the following order regarding the reducing
power of metals is correct?
(a) Ag > Cr > Mg > K (b) Ag < Cr < Mg < K
(c) Ag > Cr > K > Mg (d) Cr > Ag > Mg > K.

9. A current of 9.65 A is drawn from a Daniel cell for exactly 1 h. If molar masses of Cu and
Zn are 63.5 g mol –1 and 65.4 g mol –1 , respectively, the loss in mass at anode and gain in
mass at cathode, respectively are
(a) 11.43 g, 11.77 g (b) 11.77 g, 11.43 g
(c) 22.86 g, 23.54 g (d) 23.54 g, 22.86 g

10. In acidic medium MnO 4– is an oxidizing agent.


MnO 4– + 8H + + 5e – → Mn 2 +
If H + ion concentration is doubled, electrode potential of the half cell MnO 4– , Mn 2+ / Pt
will:
(a) increase by 28.46 mV (b) decrease by 28.46 mV
(c) increase by 14.23 mV (d) decrease by 142.30 mV.

11. The ionic mobilities of Li+, Na+, K+, Rb+ and Cs+ increase in the order:
(a) Li+, Na+, K+, Rb+ and Cs+ (b) Cs+, Rb+, K+, Na+ and Li+
+ + + + +
(c) Na , Li , Rb , Cs and K (D) None of these

12. The specific conductance of 1.0M solution a weak monobasic acid is 0.005
ohm–1 cm–1 while ionic conductances of its cation and anion are 120 and
130 ohm–1cm2mol–1 respectively. The dissociation constant of the acid is approximately.
(a) 2 × 10–3 (b) 0.02
(c) 4.0 × 10 –4
(d) None of these

13. For which of the following electrolytes, molar conductance will be three times that of
equivalence conductance at any dilution?
(a) Al2(SO4)3 (b) Na3PO4
(c) MgCl2 (d) NaCl

14. The titration curve shown in the parenthesis represents that for the:

[Type text]
[Type text]

(a) Titration of HCl vs NaOH


(b) Titration of CH3COOH vs NaOH
(c) Titration of HCOOH vs KOH
(d) Titration of HCl vs NH4OH

Conductance of
acid solution
mL of alkali added

15. The conductance that decreases with dilution is:


(a) Equivalence conductance (b) Molar conductance
(c) Specific conductance (d) Ionic conductance

16. Equal quantity of electricity are passed though three voltameters containing FeSO4,
Fe2(SO4)3 and Fe(NO3)3. Consider the following statements:
i) The amount of iron deposited in FeSO4 and Fe2(SO4)3 are equal
ii) The amount of iron deposited in Fe(NO3)3 is two third of the amount of iron deposited
in FeSO4.
iii) The amount of iron deposited in Fe2(SO4)3 and Fe(NO3)3 is equal.
Of these statements correct is/are
(a) 1 alone (b) 1 and 2
(c) 2 and 3 (d) 3 alone

17. The following facts are available: 2A– + B2 → 2B– + A2; 2C– + B2 → No reaction;
2D– + A2 → 2A– + D2. Which of the following statement is correct.
(a) E 0C− / C > E 0B− / B > E 0A− / A > E 0D− / D (b) E 0C− / C < E 0B− / B < E 0A − / A < E 0D− / D
2 2 2 2 2 2 2 2

(c) E 0
C− / C2
<E 0
B− / B2
>E 0
A − / A2
>E 0
D− / D 2
(d) E 0
C− / C2
>E 0
B− / B2
<E 0
A− / A2
<E 0
D − / D2

18. Salts of A (atomic weight 7), B (atomic weight = 27) and C (atomic weight 48) were
electrolysed under identical conditions using same quantity of electricity. It was found
that when 2.1g of A was deposited, weights of B and C deposited were 2.7g and 7.2g. The
valencies of A, B and C are respectively.
(a) 3, 1 and 2 (b) 1, 3 and 2
(c) 3, 1 and 3 (d) 2, 3 and 2

19. Cu2+ + 2e– → Cu; log[Cu2+] vs Ered graph is of the type shown in figure where OA =
0.34V, then electrode potential of the half cell of Cu/Cu2+ (0.1 M) will be

[Type text]
[Type text]

Ered
A

O
log[Cu2+]
0.059
(a) – 0.34 + V (b) 0.34 + 0.059V
2
(c) 0.34 V (d) None

20. Standard electrode potential data are useful for understanding the suitability of an oxidant
in a redox titration. Some half cell reactions and their standard potentials are given below:
MnO 4− (aq). + 8H + (aq.) + 5e − → Mn 2 + (aq.) + 4H 2 O(l) E° = 1.51 V
2− + − 3+
Cr2 O 7 (aq.) + 14H (aq.) + 6e → 2Cr (aq.) + 7H 2 O(l) E° = 1.38 V
Fe3+ (aq.) + e − → Fe 2+ (aq.) E° = 0.77 V
− −
Cl2 (g) + 2e → 2Cl (aq.) E° = 1.40 V
Identify the only incorrect statement regarding the quantitative estimation of aqueous
Fe(NO3)2
(a) MnO −4 can be used in aqueous HCl (b) Cr2 O 72 − can be used in aqueous HCl
(c) MnO −4 can be used in aqueous H2SO4 (d) Cr2 O 72 − can be used in aqueous H2SO4

21. Given E° for Cu2+ → Cu+ is +0.15 V and Cu+ → Cu is + 0.50 V


Calculate E° for Cu2+ → Cu
(a) + 0.325 V (b) + 0.125 V
(c) + 0.250 V (d) + 0.160 V

22. A solution is one molar in each of NaCl, CdCl2, ZnCl2 and PbCl2. To this, tin metal is
added. Which of the following is true? Given that,
Eo 2 + o
= – 0.126V; ESn 2 + / Sn = –0.136V
Pb / Pb

Eo = – 0.40V; EoZn 2 + / Zn = –0.763V; ENa


o
+ / Na = –2.71V
Cd 2 + / Cd

(a) Sn can reduce Na+ to Na (b) Sn can reduce Zn2+ to Zn


(c) Sn can reduce Cd2+ to Cd (d) Sn can reduce Pb2+ to Pb

23. A certain current liberates 0.5g of hydrogen in 2 hour. How many grams of copper can be
liberated by the same current flowing for the same time through a copper sulphate
solution?
(a) 12.7 grams (b) 15.9 g
(c) 31.8g (d) 63.5 g

[Type text]
[Type text]

24. The emf of the following three galvanic cells:


1. Zn/Zn2+ (1M) || Cu2+(1M)/Cu
2. Zn/Zn2+ (0.1M) || Cu2+(M) / Cu
3. Zn/Zn2+ (1M) || Cu2+(0.1M)/Cu
are represented by E1,E2,E3. Which of the following statement is true?
(a) E1 > E2 > E3 (b) E3 > E2 > E1
(c) E3 > E1 > E2 (d) E2 > E1 > E3

25. How much will the reduction potential of a hydrogen electrode change when its solution
initially at pH = 0 is neutralized to pH = 7?
(a) Increase by 0.059V (b) Decrease by 0.059V
(c) Increase by 0.41V (d) Decrease by 0.41V

SECTION – II May be more than one choice

1. SnCl2 gives a precipitate with a solution of HgCl2. In this process HgCl2


(a) is reduced (b) is oxidized
(c) acts as an oxidizing agent (d) acts as a reducing agent

2. Which of the following statements is (are) not correct?


(a) Reduction is always followed by a decrease in the oxidation number of the substance
undergoing reduction.
(b) During oxidation, a substance gains one or more electrons.
(c) A reducing agent is the substance which itself undergoes reduction
(d) The reaction C + O2 → CO2 is a redox reaction

3. Which of the following is (are) not true for a standard hydrogen electrode?
(a) The hydrogen ion conc. is 1M.
(b) H2 gas is not absorbed by platinum electrode
(c) The electrode works only as an oxidation electrode
(d E 0 + 1
H / H2 =0
2

4. Which one of the following will increase the voltage of the cell having the cell reaction
Sn + 2Ag+ → Sn2+ + Ag?
(a) Increase in the size of silver rod
(b) Increase in the concentration of Sn2+ ions
(c) Increase in the concentration of Ag+ ion
(d) Decrease in the temperature of the cell

5. The standard electrode potential of two half cells is given below.


Ag+ + e– → Ag; E0 = 0.80V
Hg 22+ + 2e − → 2Hg; E 0 = 0.78V
If the two half cells are united together

[Type text]
[Type text]

(a) silver electrode will act as anode (b) silver electrode will act as cathode
(c) mercury electrode will act as anode (d) mercury electrode will act as cathode

6. Which of the following statements is (are) correct?


(a) The reactivity of metals decreases in going down the electrochemical series.
(b) A metal can displace any other metal placed above it in the electrochemical series
from its salt solution.
(c) The oxidizing power of the substances decrease from the top to the bottom in the
electrochemical series.
(d) A redox reaction is feasible when the substance having higher reduction potential gets
reduced and the one having lower reduction potential gets oxidized.

7. The electrode potential of an electrode assembly depends upon the


(a) nature of metal and its ions (b) conc. of ions in solution
(c) temperature (d) none of the above three

8. Which of the following aqueous solution turn(s) alkaline after electrolysis?


(a) NaCl (b) CuCl2
(c) CH3COONa (d) K2SO4

9. When a nickel spatula is used to stir an aqueous copper sulphate solution


(a) the solution is stirred well without any chemical change
(b) nickel spatula gets partly oxidized into Ni2+ ions
(c) Cu2+ ions present in the solution get partly reduced into Cu
(d) copper gets deposited on the spatula

10. The E.M.F. of the following cell


Fe(s) | Fe2+ (0.1M) || Cu2+ (0.01M) | Cu
is found to be equal to 0.75V. The E.M.F. of the cell can be increased by
(a) diluting can Fe2+ solution (b) diluting Cu2+ solution
(c) increasing the conc. of Fe2+ ions (d) increasing the conc. of Cu2+

11. Which of the following reactions is (are) involved in the rusting of iron?
(a) Fe(s) → Fe2+ (aq) + 2e–
(b) O2(g) + 4H+(aq) + 4e– → 2H2O(l)
(c) 2Fe(s) + O2(g) + 4H+(aq) → 2Fe2+(aq) + 2H2O(l)
(d) Fe2+ + 2e– → Fe(s)

12. During the recharging of a discharged lead storage battery


(a) Pb changes into PbSO4 (b) PbSO4 changes into Pb and PbO2
(c) H2SO4 is consumed (d) H2SO4 is formed

13. Which of the following statements is (are) not correct?


(a) Metallic conduction is due to the movement of electrons whereas the electrolytic
conduction is due to the movement of ions.

[Type text]
[Type text]

(b) Both metallic and electrolytic conductions involve transfer of matter


(c) Electrolytic conduction decreases with rise in temperature
(d) Metallic conduction involves no chemical change

14. Which among the following statement(s) is/are correct?


(a) If E 0An + / A is negative, H+ will be reduced to H2 by the element ‘A’
(b) Compounds of (Zn, Na, Mg) are reduced by hydrogen (H2) whereas those of noble
metals (Cu, Ag, Au) are not reducible.
(c) If E 0An + / A is positive, An+ will be reduced to A by H2
(d) M | Mn+ || H+ | H2 (Pt) will be spontaneous if E 0Mn + / M is negative

15. When KCN is added to CuSO4 solution


(a) KCN acts as a reducing agent (b) KCN acts as on oxidizing agent
(c) KCN acts as a complexing agent (d) K3[Cu(CN)4] is formed

16. Electrolysis of water gives 11.2 L of hydrogen at STP at the cathode. The oxygen evolved
at the anode under similar condition is
(a) 5.6 L (b) 0.056 L
(c) 0.25 mol (d) 0.025 mol

17. The passage of electricity through certain electrolyte results in the liberation of H2 gas at
the cathode. The electrolyte could be
(a) CuCl2(aq) (b) KCl (aq)
(c) MgCl2 (aq) (d) AgNO3 (aq)

18. Electrote potential data are given below


Fe3+ (aq) + e − 
→ Fe 2 + (aq); E o = +0.77V
Al3+ (aq) + 3e − 
→ Al(s); E o = −1.66 V
Br2 (aq) + 2e − 
→ Br − (aq); E o = +1.08V
Based on the given data which statement/s is/are true ?
(a) Fe2+ is stronger reducing agent than Br− (b) Fe2+ is stronger reducing agent than Al
(c) Al is stronger reducing agent than Fe2+ (d) Br− is stronger reducing agent than Al
19. The standard emf of the cell, Cd(s)| CdCl2 (aq) || AgCl(s) | Ag(s) in which the cell
0.1M

reaction is 0.6195 V at 0°C and 0.6753 V at 25° C. The value of ∆H of the reaction at
25°C is
(a) 167.26 kJ/mol (b) −167.26 kJ/mol
(c) 40 K/mol (d) −40 Kcal/mol

[Type text]
[Type text]

20. The standard electrode potentials, E oI / I− , E oBr − / Br and E oFe / Fe2+ are respectively + 0.54
2 2

V, − 1.09 V and 0.44V as the basis of given data which of following is/are spontaneous
(a) Br2 + 2I − 
→ 2Br − + I 2 (b) Fe + Br2  → Fe 2 + + 2Br −
→ Fe 2 + + 2I −
(c) Fe + I 2  (d) I 2 + 2Br − 
→ 2I − + Br2

21. 500 cm3 of 0.01 M AgNO3 and 500 cm3 of a solution that was both 0.01 M in NaCl and
0.01M in NaBr were mixed. Which of the following statements are correct ?
Given that Ksp of AgCl and AgBr are 1.8 × 10−10 and 5.0 × 10−13 respectively.
(a) Both AgBr and AgCl are ppted out.
(b) The Ksp of AgBr is less than AgCl and so only AgBr ppted out
(c) After mixing, the concentration of Cl− is approximately equal to 0.005 M.
(d) The final concentration of Ag+ and Br− after mixing were 3.6 × 10−8 and 1.4 × 10−5
respectively.
22. The amount of ion discharged electrolysis is directly proportional to:
(a) Resistance (b) Time
(b) Current (d) Chemical equivalent of the ion
23. Which are true for a standard hydrogen electrode?
(a) The hydrogen ion concentration is 1 M
(b) Temperature is 25°C
(c) Pressure of hydrogen is 1 atmosphere
(d) It contains a metallic conductor which does not adsorb hydrogen
24. In a salt bridge, KCl fis used because
(a) it is an electrolyte
(b) KCl is found in prue crystalline state in large deposits
(c) it is a good conductor of electricity
(d) if forms a good jelly with agar-agar

25. The following four colourless salt solutions are placed in separate test tubes and strip of
Cu is placed in each. Which solution turns blue?
(a) KNO3 (b) AgNO3
(c) Zn(NO3)2 (d) ZnSO4

SECTION – III Comprehension Type Questions

Write-up I
A acidic solution of Cu+2 salt containing 0.4gm of Cu+2 is electrolysed until all the
copper is deposited. The electrolysis is continued for seven more minutes with the
volumes of solution kept at 100 ml and the current at 1.2 amp.

1. In the beginning of the electrolysis the substances evolved at cathode and anode are
respectively
(a) Cu and O2 (b) H2 and O2

[Type text]
[Type text]

(c) Cu and H2 (d) H2 and Cu

2. During another seven minutes of electrolysis the substances evolved at cathode and anode
are respectively
(a) Cu and O2 (b) H2 and O2
(c) Cu and H2 (d) Only O2 is evolved at cathode

3. The total volume of gases evolved at STP at cathode during the entire electrolysis
(a) 58.4 ml (b) 48.23 ml
(c) 60.35 ml d) 35.58 ml

4. The total volume of gases evolved at STP at anode during the entire electrolysis
(a) 58.94 ml (b) 70.44 ml
(c) 29.28 ml (d) 99.68ml

Write-up II
The standard potential of the following cell 0.23 V at and 0.21 V at
Pt, H2(g) | HCl(aq) || AgCl(s) | Ag(s)

5. The cell reaction is


1
→ Ag + Cl −
(a) H 2 + AgCl  (b) H 2 + AgCl → H + + Ag + Cl−
2
→ Ag + + 2HCl
(c) H 2 + 2AgCl  (d) H 2 + 2Cl  → 2H + + Cl2

6. The difference in the ∆G0 between the two temperature is


(a) 2000 J (b) 1930 J
(c) 2848 J (d) 3629 J

7. In the above question the value of ∆S0 is


(a) – 80.50 J (b) – 90.50 J
(c) – 96.50 J (d) –100J

8. The value of ∆H0 in the above question is


(a) 49.987 kJ (b) 50.987 kJ
(c) 48.987 kJ (d) 40.987 kJ

Write-up III
In Galvanic cell two cells are connected externally through a copper wire internally
through a salt bridge. If salt bridge is removel during the functioning of the cell then
potential drops, with the result cell can’t produce current.

9. Salt bridge possess


(a) NH4Cl in Agar Agar (b) NH4NO3 in Agar Agar
(c) KNO3 in Agar Agar (d) All the above

[Type text]
[Type text]

10. To increase the efficiency of the Galvanic cell following should be charged.
(a) Volume of anode should be increased
(b) Concentration of electrolyte around cathode should be increased
(c) Concentration of salt in the salt bridge should be increased
(d) All the above

11. The salt in the salt bridge should have following characteristics
(a) More solubility
(b) ions should have same transport number
(c) ions of salt should into interact with ions of reaction
(d) All the above

12. EMF of Galvanic cell can be calculated by


0.059 M+ 0.059 M+
(a) E = E° − log (b) E = E° + log
n M n M
.0059 M+ 2.303R M+
(c) E = E° + log (d) E = E° + log
n M n M

Write-up IV
Nernest described the effect of concentration of electrolyte on electrode potentials. The
nernest equation is given as
RT [reduced form] RT [reduced form]
E = E° – in or E = E° + in
nf [oxidised form] nf [oxidised form]
2.303 RT [oxidised form]
= E° + log
nf [reduced form]
common form of nernst equation at 25°C
0.059 [oxidised form]
E = E0 + log
n [reduced form]
+
For metal electrode reaction M n(aq) + ne − 
→ M
reduced form
oxidised form

0.059
E = E0 + log [M n + ]
n
Carefully go through the above relationships and answer the following equations.

13. A Zn rod is dipped in a solution of 0.01 M ZnSO4 solution E° for Zn+2/Zn is –0.76V. Its
electrode potential is
(a) –0.76V (b) –0.82V
(c) +0.82V (d) +0.76V

14. The hydrogen electrode is dipped in a solution of pH 3 at 25°C the potentialof the cell
would be
(a) 0.177V (b) –0.177V

[Type text]
[Type text]

(c) 0.087V (d) 0.059V

15. A cell contains two hydrogen electrodes. The negative electrode is in contact a solution of
10–6 M hydrogen ion. The EMF of the cell is 0.118V at 298k. The concentration of H+ ion
at the positive electrode is
(a) 10–5M (b) 10–6M
–4
(c) 10 M (d) 10–3M

16. Electrode potential of copper electrode where the concentration of Cu+2 I s5M is ………
V if E° Cu+2/Cu is 0.34 V.
(a) 0.36V (b) –0. 36V
(c) +0.45 V (d) –0.45 V

Write-up V
The list of standard reduction potential values arranged in the increasing order is
commonly called electrochemical series or activity series. The smaller the S.R.P. values
of are element, the greater is its ability to act as an reducing agent in the solution. The
greater the S.R.P. value of are element, the greater is its ability to out as an oxidizing
agent in solution. An element can reduce all those present under it and can oxidize all
those present above, it, in electrochemical series.
Read the following paragraph and answer the following questions.

17. Standard reduction potentials of metal electrodes X,Y and Z are + 0.34V –0.74V and
–0.4V respectively.
Which of the following is the best reducing agent?
(a) X (b) Y
(c) Z (d) None of these

18. The correct order of the reduction ability of metals


(a) X > Y > Z (b) Y > Z > X
(c) Z > X > Y (d) None of these

19. When a copper is placed in a solution of silver nitrate the solution acquires blue colour.
This is due to the formation of
(a) Cu+2 ions (b) Cu+ ions
(c) soluble complex of copper with AgNO3 (d) none of these

20. The reaction of Zn+2 + 2e– → Zn has a standard potential of –0.76V. This means
(a) Zn cannot replace hydrogen from acids (b) Zn is an oxidizing agent
(c) Zn is a reducing agent (d) Zn+2 is a reducing agent.

[Type text]
[Type text]

Answers to Assignments
SECTION - I

1. (b) 6. (a) 11. (a) 16. (c) 21. (a)


2. (d) 7. (b) 12. (c) 17. (b) 22. (d)
3. (d) 8. (b) 13. (b) 18. (b) 23. (b)
4. (c) 9. (b) 14. (d) 19. (a) 24. (d)
5. (c) 10. (a) 15. (c) 20. (a) 25. (d)

SECTION - II

1. (a), (c) 10. (a), (d) 18. (a), (c)


2. (b), (c) 11. (a), (b), (c) 19. (b), (d)
3. (b), (c) 12. (b), (d) 20. (a), (b), (c)
4. (c), (d) 13. (b), (c) 21. (a), (c), (d)
5. (b), (c) 14. (a), (c), (d) 22. (b), (c), (d)
6. (a), (d) 15. (a), (c), (d) 23. (a), (c), (d)
7. (a), (b), (c) 24. (a), (c), (d)
8. (a), (c) 16. (a), (c)
25. (b), (c), (d)
9. (b), (c), (d) 17. (b), (c)

SECTION - III

1. (a) 6. (b) 11. (d) 16. (a)


2. (b) 7. (c) 12. (a) 17. (b)
3. (a) 8. (a) 13. (b) 18. (b)
4. (d) 9. (d) 14. (b) 19. (a)
5. (b) 10. (d) 15. (c) 20. (c)

[Type text]
[Type text]

[Type text]

Potrebbero piacerti anche